You are on page 1of 66

VISIONIAS

www.visionias.in

Test Booklet Series

TEST BOOKLET

GENERAL STUDIES (P) 2024 – Abhyaas Test – 4359


C
Time Allowed: Two Hours Maximum Marks: 200

INSTRUCTIONS
1. IMMEDIATELY AFTER THE COMMENCEMENT OF THE EXAMINATION, YOU SHOULD CHECK THAT THIS
BOOKLET DOES NOT HAVE ANY UNPRINTED OR TURN OR MISSING PAGES OR ITEMS, ETC. IF SO, GET IT
REPLACED BY A COMPLETE TEST BOOKLET.

2. ENCODE CLEARLY THE TEST BOOKLET SERIES A, B, C OR D AS THE CASE MAY BE IN THE APPROPRIATE PLACE
IN THE ANSWER SHEET.

3. You have to enter your Roll Number on the Test Booklet in the Box
provided alongside. Do NOT write anything else on the Test Booklet.

4. This Test Booklet contains 100 items (Questions). Each item is printed in English. Each item comprises four
responses (answers). You will select the response which you want to mark on the Answer Sheet. In case you
feel that there is more than one correct response with you consider the best. In any case, choose ONLY ONE
response for each item.

5. You have to mark all your responses ONLY on the separate Answer Sheet provided. See direction in the
answers sheet.

6. All items carry equal marks. Attempt all items. Your total marks will depend only on the number of correct
responses marked by you in the answer sheet. For every incorrect response 1/3rd of the allotted marks will
be deducted.

7. Before you proceed to mark in the Answer sheet the response to various items in the Test booklet, you have
to fill in some particulars in the answer sheets as per instruction sent to you with your Admission Certificate.

8. After you have completed filling in all responses on the answer sheet and the examination has concluded,
you should hand over to Invigilator only the answer sheet. You are permitted to take away with you the Test
Booklet.

9. Sheet for rough work are appended in the Test Booklet at the end.

DO NOT OPEN THIS BOOKLET UNTIL YOU ARE ASKED TO DO SO

1 www.visionias.in ©Vision IAS


1. Consider the following statements about 3. Consider the following economic activities

Exchange Traded Funds (ETFs): and events:


1. Inclusion of more economic activities
1. They generally boast lower expense ratios
under priority sector lending (PSL) norms
compared to mutual funds. by RBI
2. They offer increased flexibility and 2. Introduction of stringent KYC norms by

liquidity compared to mutual funds. banks to control loan defaults


3. Promotion of financial literacy in rural
3. They generally experience lower levels of
India
short-term price fluctuations compared to 4. Closure of bank branches in economically
mutual funds. unviable areas

How many of the above are advantages of How many of the above economic activities
and events may promote financial inclusion?
Exchange Traded funds (ETFs)?
(a) Only one
(a) Only one (b) Only two
(b) Only two (c) Only three
(d) All four
(c) All three

(d) None
4. With reference to Household Consumption
Expenditure Survey (HCES) 2022-23,
2. Consider the following statements with consider the following statements:
1. It is conducted by the Ministry of
reference to the Payment and Settlement
Consumer Affairs every five years.
Systems (PSS) Act, 2007:
2. The rate of increase in rural India's
1. The Act empowers the Central Monthly Per Capita Consumption
Government to designate payment systems Expenditure (MPCE) has been more
compared to urban India's MPCE since
in India.
2011-12.
2. The Act prohibits foreign entities from
3. The survey indicates a decrease in the
operating a payment system in India. percentage share of expenditure on food
Which of the statements given above is/are items by rural households since 2011-12.
How many of the above statements are
correct?
correct?
(a) 1 only
(a) Only one
(b) 2 only (b) Only two
(c) Both 1 and 2 (c) All three
(d) None
(d) Neither 1 nor 2
www.visionias.in 2 ©Vision IAS
5. Consider the following statements: 7. In business and economic parlance, the terms
1. Aluminium wagon rakes are 100 percent Cheetah and Gazelle, sometimes seen in news,
recyclable. refer to:
2. The manufacturing cost of aluminium (a) shares expected to provide high returns in
wagon rakes is lower than conventional the future.
(b) developing countries having rapid
rakes.
economic growth rate.
3. Aluminium wagon rakes have a potential
(c) startups expected to attain unicorn status
to reduce Indian Current Account Deficit
soon.
(CAD).
(d) sunrise industries having high potential for
4. The carbon footprint of Aluminium wagon employment generation.
rakes is lower than conventional rakes.
5. Aluminium wagon rakes can carry more 8. Consider the following statements regarding
payload per trip over conventional rakes. Five Year Plans (FYPs):
How many of the statements given above are 1. India did not achieve its growth target in
correct? the first two Five Year Plans (FYPs).
(a) Only two 2. The objective of the Eighth FYP was
'Towards Faster, and More Inclusive
(b) Only three
Growth'.
(c) Only four
3. The Twelfth FYP reduced the number of
(d) All five
centrally sponsored schemes in order to
improve their effective implementation.
6. Consider the following statements regarding How many of the statements given above are
Gross Domestic Product (GDP) and Gross correct?
Value Added (GVA): (a) Only one
Statement I: Growth rate of a country's (b) Only two
economy can vary depending on whether one (c) All three
measures GDP or GVA. (d) None
Statement II: Growth in tax collection and a
9. Consider the following themes:
reduction in subsidies can inflate GDP
1. Counter the rising presence of China in the
numbers even when GVA remains the same.
Indian Ocean Region
Which one of the following is correct in
2. Develop deep sea mining in the Indian
respect of the above statements?
Ocean Region
(a) Both Statement-I and Statement-II are 3. Promote ocean, coastal and river cruise
correct and Statement-II is the correct sector
explanation for Statement-I 4. Become a global player in shipbuilding,
(b) Both Statement-I and Statement-II are repair and recycling
correct and Statement-II is not the correct How many of the above themes are included
explanation for Statement-I in the Amrit Kaal Vision 2047 Action Plan
(c) Statement-I is correct but Statement-II is developed by the government of India?
incorrect (a) Only one
(b) Only two
(d) Statement-I is incorrect but Statement-II is
(c) Only three
correct
(d) All four
www.visionias.in 3 ©Vision IAS
10. Which inflation index is used to adjust the 13. With reference to the Governor’s address to
purchase price of capital assets on the basis of the state legislature consider the following
inflation? statements:
(a) Consumer Price Index (Urban) 1. The Constitution gives the Governor the
(b) Wholesale Price Index power to address the first sitting of the
(c) Cost Inflation Index legislature each year.
(d) Consumer Price Index- Industrial Workers 2. The Office of the Governor prepares the
governor’s speech with inputs from the

11. The Directive Principle of State Policy that council of ministers.

mandates the state to organize agriculture and 3. The Constitution of India mandates that

animal husbandry on modern and scientific the Governor cannot deviate from the text

lines is based on which of the following of the speech.

philosophical principles? How many of the above statements are not

(a) Anarchism correct?


(a) Only one
(b) Nihilism
(b) Only two
(c) Conservatism
(c) All three
(d) Progressivism
(d) None

12. Consider the following statements:


14. Consider the following pairs:
Statement-I: The Supreme Court can not
Commission/ Concerned with
deviate from its previous judgements.
Committee
Statement-II: Supreme Court is a court of
1. Rajamannar : Centre-state fiscal
record and its judicial proceedings are
Committee relationship
recognized as legal precedence.
Which one of the following is correct in 2. Justice G. Rohini : Prison reforms
Commission
respect of the above statements?
(a) Both Statement-I and Statement-II are 3. Justice Mulla : OBC

correct and Statement-II is the correct Committee subcategorization

explanation for Statement-I 4. Ranganath : Religious and

(b) Both Statement-I and Statement-II are Mishra Commission Linguistic

correct and Statement-II is not the correct Minorities

explanation for Statement-I How many of the above pairs are correctly

(c) Statement-I is correct but Statement-II is matched?

incorrect (a) Only one

(d) Statement-I is incorrect but Statement-II is (b) Only two


(c) Only three
correct
(d) All four
www.visionias.in 4 ©Vision IAS
15. Which among the following envisages a short- 18. With reference to appointment of the Director
term temporary release of a prisoner without General of Police (DGP) of a State, consider
any reason, after a stipulated number of years the following statements:
have been served in prison although the
1. Union Public Service Commission sets
prisoner does not have an absolute legal right
the guidelines for empanelling officers for
to claim it?
(a) Reprieve appointment as State DGP in India.

(b) Parole 2. Recent guidelines consider only officers


(c) Furlough with at least six months of service left
(d) Respite before retirement for DGP appointment.
3. The recent guidelines have reduced the
16. In the context of the Indian Parliament the
minimum experience for DGP eligibility
term “Naming a Member” refers to:
from 30 to 25 years.
(a) It is a type of pecuniary punishment
administered by the Presiding Officer of a How many of the statements given above are

legislature to an offender for breach of correct?


privilege or contempt of the House. (a) Only one
(b) Immediately after the Question Hour, a (b) Only two
named Member may address an urgent (c) All three
public issue with the Chairman's
(d) None
permission.
(c) A Member against whom comments or
19. Recently, the Parliament passed the
criticism of a personal nature are made on
the floor of the House is entitled to make, Constitution (106th Amendment) Act, 2023, to
with the consent of the Chairman, a provide for one-third reservation to women.
personal explanation in his defence. This act provides women reservation in which
(d) The Chairman alerts the House to a of the following legislative bodies?
member's misconduct, aiming to suspend
1. Rajya Sabha
them from service for disregarding the
2. Lok Sabha
Chair's authority or willfully obstructing
3. State Legislative assembly
House business.
4. State Legislative council
17. In which of the following cases, Rajya Sabha Select the correct answer using the code given
has more powers than Lok Sabha? below.
(a) Passing of the Money Bill (a) 1, 2, and 3 only
(b) Passing of a Constitutional Amendment
(b) 1 and 2 only
Bill
(c) 2 and 3 only
(c) Election of the President
(d) Removal of the Vice President (d) 1 and 4 only

www.visionias.in 5 ©Vision IAS

Telegram: UPSC_SARTHI
20. With reference to the Chief Election 22. Consider the following statements regarding

Commissioner (CEC) and other Election the insolation received on the earth:

Commissioners (ECs), consider the following 1. Across all latitudes, the equator receives

statements: the maximum insolation.

1. The Prime Minister shall appoint the CEC 2. Generally, at the same latitude, the

and other ECs on the recommendation of a insolation is more over the oceans than on
continents.
Selection Committee.
Which of the statements given above is/are
2. The Selection Committee consists of the
correct?
Chief justice of India, the Leader of the
(a) 1 only
Opposition, and a Union Cabinet Minister.
(b) 2 only
3. CEC and other ECs are not eligible for
(c) Both 1 and 2
reappointment.
(d) Neither 1 nor 2
How many of the statements given above are

correct?
23. In mountainous areas, the decrease in
(a) Only one
temperature with increasing altitude leads to
(b) Only two
the corresponding change in natural
(c) All three
vegetation. In this context, which of the
(d) None
following is the correct sequence of natural
vegetation zones in the ascending order of
21. With reference to the 2011 census, arrange the altitude?
following states in decreasing order in terms of (a) Temperate grasslands - Wet temperate
Scheduled Tribe population. evergreen broad-leaf forest - Alpine
1. Andhra Pradesh vegetation - Mosses and lichens
2. Madhya Pradesh (b) Temperate grasslands - Wet temperate

3. Nagaland evergreen broad-leaf forest - Mosses and

4. Uttar Pradesh lichens - Alpine vegetation

Select the correct answer using the code given (c) Wet temperate evergreen broad-leaf forest

below. - Temperate grasslands - Mosses and

(a) 1-2-3-4 lichens - Alpine vegetation

(b) 2-1-3-4 (d) Wet temperate evergreen broad-leaf


forest - Temperate grasslands - Alpine
(c) 2-1-4-3
vegetation - Mosses and lichens
(d) 1-3-4-2
www.visionias.in 6 ©Vision IAS

Telegram: UPSC_SARTHI
24. Which of the following statements best 26. With reference to the formation of the rock
describes the formation of 'cavum clouds'? system, arrange the following rock system in
chronological order:
(a) They are formed when aircraft pass
1. Cuddapah rock system
through layers of altocumulus clouds.
2. Dravidian rock system
(b) They are formed due to the intense 3. Dharwar rock system
convection associated with the 4. Gondwana rock system

Intertropical Convergence Zone (ITCZ), Select the correct answer using the code given
below.
resulting in the formation of large cloud
(a) 2-1-3-4
holes.
(b) 4-1-3-2
(c) They are formed due to the condensation (c) 1-2-4-3
of water vapor at high altitudes, creating (d) 3-1-2-4

visible formations in the sky.


27. Consider the following statement regarding the
(d) They are formed as a result of the Polar
asthenosphere:
Vortex interacting with atmospheric 1. This layer is the main source of magma.
moisture, causing localized cloud 2. The density of the asthenosphere is lower
disruptions. than that of the lithosphere.
3. The ductility of the asthenosphere is
higher than that of the lithosphere.
25. Consider the following statements regarding
How many of the statements given above are
minerals found in India: correct?
1. Some riverbeds contain rare metal (a) Only one
deposits. (b) Only two
(c) All three
2. Some river basins contain shale gas
(d) None
deposits.
3. Some coastal areas have crude oil 28. A waste management technology, BioTRIG,
reserves. seen in the news recently is:

4. Some coastal areas have ilmenite and (a) a process of conversion of organic waste
to energy in the presence of oxygen.
rutile deposits.
(b) an in-situ method of bioremediation
How many of the statements given above are
wherein pressurized air is injected to
correct? trigger microbial decomposition.
(a) Only one (c) a new technology that allows pyrolysis at

(b) Only two a community level.


(d) an ex-situ method of bioremediation in
(c) Only three
which contaminants and pollutants are
(d) All four converted into water and harmless gases.
www.visionias.in 7 ©Vision IAS

Telegram: UPSC_SARTHI
29. Consider the following statements: 31. Consider the following statements with
Statement I: Polar bears can only survive in reference to Dhanauri Wetlands:
1. It is a natural wetland which has been
cold arctic temperatures and cannot tolerate
recognised as an Important Bird Area by
warmer temperatures.
Bird Life International.
Statement II: Polar bears are eurythermal 2. It has been recognised as a wetland of
animals. International importance under the Ramsar
Which one of the following is correct in convention.
3. The river Yamuna directly feeds Dhanauri
respect of the above statements?
Wetland.
(a) Both Statement-I and Statement-II are
How many of the statements given above are
correct and Statement-II is the correct correct?
explanation for Statement-I (a) Only one
(b) Both Statement-I and Statement-II are (b) Only two
(c) All three
correct and Statement-II is not the correct
(d) None
explanation for Statement-I
(c) Statement-I is correct but Statement-II is 32. Consider the following:
incorrect 1. Plastic Packaging
(d) Statement-I incorrect but Statement-II is 2. Batteries
3. Tyres
correct
4. Printers
5. Waste oil
30. Consider the following statements with In India, extended producer responsibility
reference to regulation of persistent organic (EPR) is applicable to manufacturers/
pollutants (POPs): producers of how many of the above given
products?
1. The use of DDT has been stopped
(a) Only two
completely in India for both agricultural (b) Only three
and non agricultural use. (c) Only four
2. India banned use of endosulfan which also (d) All five
has bio-accumulation properties.
33. Consider the following organisms:
3. Basel convention does not ban exports of
1. Skunk
hazardous waste to developing countries 2. Vulture
for the purpose of recycling. 3. Bombardier Beetle
How many of the statements given above are How many of the above organisms uses odor
correct? or strong smell as a natural defense
mechanism against predators?
(a) Only one
(a) Only one
(b) Only two (b) Only two
(c) All three (c) All three
(d) None (d) None
www.visionias.in 8 ©Vision IAS

Telegram: UPSC_SARTHI
34. Which of the following Acts provided for the 36. Consider the following statements:
Statement-I: In 2024, the great lake region
‘compensatory afforestation’ for the first time
recorded the lowest levels of ice.
in India?
Statement-II: Global warming, La Nina and
(a) Forest Conservation Act, 1980 frequent blasts of Arctic air have led to high

(b) Forest Rights Act, 2006 temperature in the region.


Which one of the following is correct in
(c) Biological Diversity Act, 2002
respect of the above statements?
(d) Compensatory Afforestation Fund Act, (a) Both Statement-I and Statement-II are

2016 correct and Statement-II is the correct


explanation for Statement-I.
(b) Both Statement-I and Statement-II are
35. With reference to the Diverse Epigenetic correct and Statement-II is not the correct

Epidemiology Partnership (DEEP) project, explanation for Statement-I.


(c) Statement- I is correct but Statement -II is
consider the following statements:
incorrect.
1. The DEEP project, spanning five years, is (d) Statement-I is incorrect but Statement-II is

led by the World Health Organization correct.

(WHO) and Bill & Melinda Gates


37. Which of the following statements best
Foundation. describes the term ‘Internal Carbon Pricing’?
2. It will help to focus on the genetic aspects (a) It is the cost of emissions that the public
pays for and ties them to their sources
of Non-Communicable Diseases (NCDs).
through a price usually in the form of a
3. The project involves generating genomic
price on the carbon dioxide emitted.
datasets and analyzing DNA methylation (b) It is the value a company voluntarily sets
to internalize the economic cost of its
data.
greenhouse gas emissions.
How many of the above statements are
(c) It is a type of tax which directly sets a
correct? price on green house gas emissions based

(a) Only one on the carbon content of fossil fuels.


(d) It is used to calculate the monetary
(b) Only two
compensation that developed countries
(c) All three give to developing countries under the

(d) None Kyoto Protocol.

www.visionias.in 9 ©Vision IAS

Telegram: UPSC_SARTHI
38. Consider the following statements with 40. The Kew International Medal, Emerald Award
reference to Pradhan Mantri Matru Vandana and Tyler Prize are given for the contribution
Yojana (PMMVY): in which among the following fields?
1. The scheme aims to provide a cash (a) Environment
incentive to all mothers who give first live (b) Sports

birth and for the second live birth only if it (c) Economics

is a girl child. (d) Mathematics

2. The scheme seeks to discourage pre-birth


41. With reference to the Tebhaga movement of
sex selection.
the 1940s, consider the following statements:
Which of the statements given above is/are
1. It was led by the Bengal Provincial Kisan
correct?
Sabha and actively supported by the
(a) 1 only
Rashtriya Swayamsevak Sangh.
(b) 2 only
2. The movement witnessed an energetic
(c) Both 1 and 2
participation of women and the formation
(d) Neither 1 nor 2 of Nari Bahinis.
3. The Bargadar Bill proposed to concede the
39. With reference to ASHA, Anganwadi workers demands of the peasants was readily
and Auxiliary Nursing Midwifery (ANM), accepted by the Congress and was
consider the following statements: successfully enacted.
1. Anganwadi workers provide primary How many of the statements given above are
health care assistance and offer correct?
community-level curative assistance for (a) Only one
minor ailments like diarrhea and fevers. (b) Only two
2. ASHA workers are voluntary workers (c) All three

entrusted with providing health check- (d) None

ups, immunization and pre-school


42. On May 4th, 1930, Mahatma Gandhi was
informal education to children.
arrested for announcing that he would lead a
3. The role of ANM is to provide healthcare
raid on Dharasana Salt Works on the West
assistance only for non communicable
Coast. Which of the following measures was
disease along with traditional
not sanctioned by the Congress Working
Reproductive and Child Health centric
Committee after Mahatma Gandhi's arrest?
functions.
(a) No-chowkidari-tax campaign in Zamindari
How many of the statements given above are
areas
correct? (b) Non payment of revenue in Ryotwari areas
(a) Only one (c) Violation of forest laws in the Central
(b) Only two Provinces
(c) All three (d) Voluntary resignations from Provincial
(d) None Legislative Assemblies
www.visionias.in 10 ©Vision IAS

Telegram: UPSC_SARTHI
43. Consider the following events: 46. In the context of the cultural history of India,
1. Backup naval forces were dispatched from the terms pachisi, chaturanga and gyan
present-day Mauritius. chaupar refer to
2. Captain Paradise's small army defeated the
(a) spices
army of Mahfuz Khan.
(b) board games
3. Robert Clive was imprisoned.
(c) books
Which of the following conflicts has been
referred to in the statements given above? (d) religious sects

(a) Battle of Plassey


(b) The First Carnatic War 47. Consider the following statements regarding
(c) The Second Carnatic War the UNESCO site Champaner-Pavagadh
(d) Battle of Buxar Archeological Park:
1. It is home to religious structures of Hindu,
44. He was classified as one of the leading
Jain, and Muslim.
moderate leaders of the Indian National
2. Several water-harvesting installations in
Congress. He had famously stated 'What
the form of lakes, ponds, wells, stepwells,
Bengal thinks today, India thinks tomorrow'.
The English daily 'The Hitavada' was founded and tanks were employed at this site.

by him in 1911. 3. Mahmud Begarha of Gujarat Sultanate had


Who among the following has been described set up a coin mint at Pavagadh.
in the passage given above? How many of the statements given above are
(a) Gopal Krishna Gokhale correct?
(b) Surendranath Banerjee
(a) Only one
(c) Shyama Prasad Mukherjee
(b) Only two
(d) Dadabhai Naoroji
(c) All three
(d) None
45. Consider the following pairs:
GI Tag Product State
1. Karuppur kalamkari : Tamil Nadu 48. Iberian Style of architecture, though not
2. Idu Mishmi textile : Meghalaya prevalent in India, has influenced modern
3. Lakadong turmeric : Arunachal Indian architecture to some extent. The
Pradesh architecture had its origin in which of the
How many of the above pairs are correctly
following regions?
matched?
(a) Russia and Ukraine
(a) Only one
(b) Denmark and the Netherlands
(b) Only two
(c) All three (c) Turkey and Syria

(d) None (d) Portugal and Spain


www.visionias.in 11 ©Vision IAS

Telegram: UPSC_SARTHI
49. Consider the following statements in respect 51. Consider the following:
of the 19th Asian Games 2022: 1. New Look Policy

1. Only four countries won over 100 medals. 2. Look East policy

2. This edition of the Asian Games marked 3. Act East Policy

the inaugural inclusion of cricket as a 4. Neighbourhood first policy

featured sport. How many of the above are P.V. Narasimha


Rao's contributions to India's foreign policy?
3. Shooting fetched India the most number of
(a) Only one
gold medals.
(b) Only two
How many of the statements given above are
(c) Only three
correct?
(d) All four
(a) Only one
(b) Only two
52. 'Voice of the Global South Summit' was
(c) All three
recently seen in the news in the context of
(d) None
(a) World Trade Organization
(b) Association of Southeast Asian Nations
50. Consider the following statements about sports (c) G20
awards: (d) United Nations
1. Maulana Abul Kalam Azad Trophy is
given to a university for top performance 53. In the midst of Israel’s ongoing military
in inter-university tournaments over the attacks on the Gaza Strip, the United Nations
last one year. Secretary-General has invoked Article 99 of

2. The Arjuna Award is given for the UN Charter. In this context, consider the

consistently good performance over a following statements:

period of four years. 1. The adoption of the resolution requires

3. Olympic bronze medal-winning approval from at least nine member states,


with no vetoes from permanent members.
weightlifter Karnam Malleswari was the
2. Article 99 does not lead to immediate
first Indian woman to receive the Arjuna
direct action from the UN or the Security
award.
Council.
How many of the statements given above are
Which of the statements given above is/are
correct?
correct?
(a) Only one
(a) 1 only
(b) Only two
(b) 2 only
(c) All three
(c) Both 1 and 2
(d) None (d) Neither 1 nor 2

www.visionias.in 12 ©Vision IAS

Telegram: UPSC_SARTHI
54. Consider the following statement with regard 56. Consider the following statements regarding
to the rights of coastal states in Exclusive
‘E Ink’ displays technology recently seen in
Economic Zone:
1. A coastal state has a sovereign right for the news:
the exploration and exploitation of natural
1. It is a type of e-paper display that uses
resources, whether living or non-living.
2. A coastal state has sovereign rights for charged particles to create text and images
production of energy from the water,
on display.
current and winds.
3. A coastal state has no right to establish 2. Like LCD, it uses a backlight display
artificial islands.
instead of reflected light which improves
4. A coastal state has the right to carry out
marine scientific research. readability under bright light.
How many of the statements given above are
Which of the statements given above is/are
correct?
(a) Only one correct?
(b) Only two
(a) 1 only
(c) Only three
(d) All four (b) 2 only

(c) Both 1 and 2


55. In the context of the recently inaugurated
National Automated Fingerprint Identification (d) Neither 1 nor 2
System (NAFIS) in India, consider the
following statements:
1. It is a centralized database of fingerprints 57. It is a rare element widely used in the
developed by the National Crime Records electronics and semiconductor industry
Bureau (NCRB) to facilitate the quick and
easy disposal of criminal cases. because of its unique characteristics. It is part
2. It assigns a unique 10-digit National of the refractory metals group and is very hard,
Fingerprint Number (NFN) for each
criminal, based on biometrics. ductile, and lustrous. Recently, a team of
3. The unique ID, assigned for a lifetime, researchers from India has discovered the
ensures that different crimes under various
FIRs are linked to the same NFN. presence of it in the Sutlej River.
4. Uttar Pradesh became the first state in Which element has been referred to in the
India to identify a deceased person
through NAFIS. above description?
How many of the above statements are (a) Silicon
correct?
(a) Only one (b) Tantalum
(b) Only two (c) Germanium
(c) Only three
(d) All four (d) Gallium

www.visionias.in 13 ©Vision IAS

Telegram: UPSC_SARTHI
58. Consider the following statements regarding 61. Which of the following subjects is not a part
End-to-End encryption technology: of concurrent list under the seventh schedule
1. In end-to-end encryption, decryption keys to the Constitution of India?
are typically stored on a central server
(a) Marriage and divorce
managed by the service provider.
(b) Industrial and labour disputes
2. It is only used for instant messaging
(c) Bankruptcy and insolvency
applications like WhatsApp and iMessage.
(d) Public health and sanitation
3. It protects both the data and the metadata.
How many of the above statements are
correct? 62. Which of the following statements best
(a) Only one describes the concept of negative liberty?
(b) Only two (a) The freedom to pursue one's own goals
(c) All three
and interests
(d) None
(b) The absence of external constraints or
interference
59. The terms Bluejacking, Bluesnarfing, and
(c) Exercising liberty to pursue negative goals
Juice jacking sometimes mentioned in the
news are related to: (d) The right to equal treatment under the law

(a) Cyberattack
(b) Generative AI 63. Consider the following statements regarding
(c) Goldilocks Zone Parliamentary standing committees:
(d) Maritime piracy 1. Parliamentary standing committees are
permanent committees constituted by the
60. Consider the following applications of
Parliament.
biotechnology:
2. The recommendations made by
1. Minimally invasive screens for disease-
related genetic abnormalities parliamentary standing committees are

2. Screening pregnancies for trisomy 21 binding on the government.


3. Chromosome microarray analysis (CMA) 3. The members of parliamentary standing
4. Assessment of allograft injury or rejection committees are elected by the respective
5. Liquid biopsy Houses of Parliament.
6. Non-invasive prenatal testing.
How many of the statements given above are
How many of the above applications involve
correct?
cfDNA technology?
(a) Only one
(a) Only three
(b) Only two
(b) Only four
(c) Only five (c) All three

(d) All six (d) None


www.visionias.in 14 ©Vision IAS

Telegram: UPSC_SARTHI
64. The term ‘Grassroots Innovation Forum’ often 67. Consider the following statements:
appears in the news in the context of the 1. It operates within the framework of the
affairs of a group of countries known as United Nations.
(a) European Union 2. It was formed in response to the aftermath
(b) Shanghai Cooperation Organization of the Second World War.
(c) Association of Southeast Asian Nations 3. Its headquarter is located in Geneva,
(d) G20 Switzerland.
4. India is currently its member.
65. Recently, the Parliament passed the Press and Which of the following institutions/
Registration of Periodicals Act, 2023. In this organisations has been best described by the
context, consider the following:
statements given above?
1. Newspapers
(a) International Court of Justice
2. Books
(b) Food and Agriculture Organization
3. Scientific Journals
(c) International Organization for Migration
4. Comment on public news
(d) International Labour Organization
How many of the above are covered under the
Act?
68. With reference to Judicial initiatives in India,
(a) Only one
consider the following statements about the
(b) Only two
Legal Aid Defense Counsel System (LADCS):
(c) Only three
1. It is a project executed by the National
(d) All four
Legal Services Authority (NALSA).
2. It offers free legal aid and assistance to
66. With reference to parliamentary tools,
eligible individuals in civil and criminal
consider the following statements about the
matters.
point of order:
3. Lawyers involved in LADCS are
1. It can be raised by any member
prohibited from taking on any private
irrespective of his/her party affiliation.
cases.
2. No debate is allowed on a point of order.
How many of the statements given above are
Which of the statements given above is/are
correct?
correct?
(a) 1 only (a) Only one

(b) 2 only (b) Only two

(c) Both 1 and 2 (c) All three

(d) Neither 1 nor 2 (d) None

www.visionias.in 15 ©Vision IAS

Telegram: UPSC_SARTHI
69. Consider the following policies: 71. Consider the following:
1. Increasing the money supply 1. Repo rate
2. Reducing the tax rates
2. 91 day T-bill yield
3. Raising interest rates
3. 182 day T-bill yield
4. Investing in large capital projects by the
4. Benchmark market interest rate as
government
How many of the above are considered as the developed by the Financial Benchmarks

reflation policies? India Pvt. Ltd.


(a) Only one
How many of the above benchmarking
(b) Only two
mechanisms are eligible for the External
(c) Only three
Benchmark Lending Rate?
(d) All four
(a) Only one

70. Consider the following statements regarding (b) Only two


the framework for acceptance of Green (c) Only three
Deposits in India:
(d) All four
1. It is applicable to all deposits taking Non-
Banking Financial Companies registered
72. Consider the following:
with RBI, Scheduled Commercial Banks
and Small Finance Banks. 1. Acquisition of existing tangible fixed

2. Investments made by a bank in Sovereign assets


Green Bonds are covered under the
2. Acquisition of new intangible fixed
framework.
assets
3. Green projects financed under the
3. Improvements to tangible non-produced
framework can be classified under the
priority sector if they meet priority sector assets

lending guidelines of RBI. How many of the above are ways of gross

How many of the statements given above are capital formation?


correct?
(a) Only one
(a) Only one
(b) Only two
(b) Only two
(c) All three
(c) All three
(d) None (d) None

www.visionias.in 16 ©Vision IAS

Telegram: UPSC_SARTHI
73. Consider the following statements regarding 75. Consider the following types of deposits:
Agreement on Trade-Related Investment 1. Recurring deposits
Measures (TRIMs): 2. Inter-bank deposits
1. It applies to measures that affect trade in 3. Deposits of foreign governments
goods and services. 4. Deposits of State governments
2. It prohibits certain measures that violate How many of the above are insured by
the national treatment and quantitative Deposit Insurance and Credit Guarantee
restrictions requirements. Corporation (DICGC)?
3. All WTO member countries are parties to (a) Only one
this Agreement. (b) Only two
How many of the above statements are (c) Only three
correct? (d) All four
(a) Only one
(b) Only two 76. In the context of modern Indian history,
(c) All three Reshmi Rumal Tehreek (or Silk Letter
(d) None Movement) was
(a) A branch of the Aligarh movement that
74. Consider the following criteria: supported English education of Muslim
1. Crude oil and/or product reserves women.
equivalent to 90 days of the previous (b) The challenge of Deobandi leaders to the
year’s net imports, to which the dominance of the British Empire.
government has immediate access. (c) The protest of cotton growers of rural
2. A demand restraint programme to reduce India against the import of Chinese silk.
national oil consumption by up to 10%. (d) A secret movement led by Ashfaqulla
3. Legislation and measures to ensure that all Khan.
oil companies under its jurisdiction report
information upon request. 77. The Indian National Congress (INC) came into
4. Must be a member of Group of 20. being in 1885 with Womesh Chandra
How many of the above criterias are required Bonnerjee as its first President. In this context,
for getting full membership in the who among the following served as the first
International Energy Agency (IEA)? General Secretary of the INC?
(a) Only one (a) Lord Dufferin
(b) Only two (b) Womesh Chandra Bonnerjee
(c) Only three (c) A.O. Hume
(d) All four (d) Dadabhai Naoroji

www.visionias.in 17 ©Vision IAS

Telegram: UPSC_SARTHI
78. In the context of modern India, consider the 81. With refrence to the Gupta period, the terms
following statements regarding the Bengal aprahata and sadbala refer to
Volunteers Corps: (a) government officials
1. It was set up under the leadership of (b) types of agricultural lands
Kamaladevi Chattopadhyay during the (c) coins
1928 Calcutta Congress session. (d) military camps

2. It was set up mainly to demand voting


82. Consider the following statements regarding
rights for women.
slavery during the Delhi Sultanate period :
Which of the statements given above is/are
1. Balban prohibited owning slaves to raise
correct?
the prestige of the Sultanate.
(a) 1 only
2. During the reign of Alauddin Khalji, the
(b) 2 only
prices of the slaves were fixed.
(c) Both 1 and 2
Which of the statements given above is/are
(d) Neither 1 nor 2 correct?
(a) 1 only
79. Consider the following dynasties: (b) 2 only
1. Ikshvakus (c) Both 1 and 2
2. Salankayanas (d) Neither 1 nor 2
3. Kadambas
4. Abhiras 83. Sri Krishna Deva Raya, the famous ruler of the
5. Karakota Vijayanagar Empire ruled for nearly 20 years.
How many of the above dynasties were He was rated by his contemporaries as a
established in the South Indian region soon powerful ruler. In this context, who among the
after the decline of Satavahanas? following was his contemporary?
(a) Only two (a) Maharana Pratap
(b) Babur
(b) Only three
(c) Jahangir
(c) Only four
(d) Akbar
(d) All five

84. The traditional theatre form mainly performed


80. Consider the following pairs:
in the Kashmir region, is a unique combination
Festival State
of dance, music, and acting. Satire, wit, and
1. Madai Mela : Rajasthan
parody are preferred for inducing laughter. In
2. Karaga : Karnataka this theatre form, music is provided with
3. Moatsu : Sikkim surnai, nagaara and dhol. The above
How many of the pairs given above are description depicts which of the following
correctly matched? traditional theatre?
(a) Only one (a) Bhaona
(b) Only two (b) Dashavatar
(c) All three (c) Bhand Pather
(d) None (d) Maach
www.visionias.in 18 ©Vision IAS

Telegram: UPSC_SARTHI
85. Consider the following statements regarding 87. Consider the following:
paintings of the Mughal period: 1. Chital
1. During Jahangir's period paintings had 2. Musk Deer
broad margins which were gorgeously 3. Barasingha
decorated with flora and faces of human
4. Caribou
figures.
5. Mouse Deer
2. Mir Syed Ali was a painter commissioned
6. Barking Deer
by Babur to illustrate his book
How many of the above given species of deer
Baburnama.
are naturally found in India?
3. The famous illustrated work of
Hamzanama was done during the reign of (a) Only three

Shah Jahan. (b) Only four


How many of the statements given above are (c) Only five
correct? (d) All six
(a) Only one
(b) Only two 88. Which one of the following does not belong to
(c) All three the phylum of the other three species?
(d) None
(a) Octopus
(b) Snail
86. Recently, telecom giant Airtel, introduced
(c) Crab
recycled Polyvinyl chloride (PVC) SIM cards,
(d) Oyster
to combat plastic pollution. In this context,
consider the following statements with
reference to recycling of various types of 89. With reference to the Indian biodiversity,

plastics: consider the following pairs:


1. The process of recycling High Density Types of species Example
Polyethylene is more difficult as compared 1. Keystone species : Bees
to Low Density Polyethylene. 2. Precinctive species : Lion-tailed
2. Polystyrene is the most common type of macaque
plastic to be recycled in the world.
3. Indicator species : Frogs
3. Recycling of PVC is more economically
4. Invasive alien : Prosopis julifora
feasible then recycling Polyethylene
species
terephthalate (PET).
How many pairs given above are correctly
How many of the statements given above are
matched?
correct?
(a) Only one (a) Only one

(b) Only two (b) Only two


(c) All three (c) Only three
(d) None (d) All four

www.visionias.in 19 ©Vision IAS

Telegram: UPSC_SARTHI
90. With reference to carbon cycle, consider the 92. Recently, Government of India has released a
following statements: report on the 'Status of Leopards in India'. In
1. Carbon is the most abundant element in this context, consider the following statements
the earth's crust. regarding population of leopards in India:
2. On Earth, most carbon is stored in the 1. Maharashtra has the highest number of
ocean and atmosphere rather than in rocks leopards in India.
and sediments. 2. More than half of the total leopard
3. The amount of carbon in the earth's carbon population is concentrated in the states of
cycle is increasing due to excessive use of Madhya Pradesh, Maharashtra, Karnataka
fossil fuels. and Tamil Nadu.
How many statements given above are 3. All Himalayan states have shown a steady
correct? rise in the population of leopards in the
(a) Only one last five years.
(b) Only two How many of the statements given above are
(c) All three
correct?
(d) None
(a) Only one
(b) Only two
91. Consider the following statements:
(c) All three
Statement I: Global dimming reduces the
(d) None
amount of sun’s rays reaching the earth’s
atmosphere causing a drop of temperatures
93. Consider the following statements regarding
around the globe.
prokaryotic and eukaryotic cells:
Statement II: Global dimming occurs due to
1. Prokaryotic cells lack a true nucleus, while
changes in the sun's luminosity.
eukaryotic cells have a membrane-bound
Which one of the following is correct in
nucleus.
respect of the above statements?
2. Eukaryotic cells are typically smaller in
(a) Both Statement-I and Statement-II are
size as compared to prokaryotic cells.
correct and Statement-II is the correct
3. Unlike the eukaryotic cells, the
explanation for Statement-I
prokaryotic cells do not have DNA.
(b) Both Statement-I and Statement-II are
How many of the statements given above are
correct and Statement-II is not the correct
correct?
explanation for Statement-I
(a) Only one
(c) Statement-I is correct but Statement-II is
(b) Only two
incorrect
(c) All three
(d) Statement-I incorrect but Statement-II is
(d) None
correct

www.visionias.in 20 ©Vision IAS

Telegram: UPSC_SARTHI
94. Consider the following statements: 96. Which of the following states has the lowest
Statement-I: While traveling from the equator drainage basin area of river Ganga?
towards the poles the weight of a person (a) Madhya Pradesh
increases. (b) Uttar Pradesh
Statement-II: The coriolis force increases (c) West Bengal
while moving from the equator to the poles.
(d) Rajasthan
Which one of the following is correct in
respect of the above statements?
97. Arrange the following straits of the world from
(a) Both Statement-I and Statement-II are
west to east:
correct and Statement-II is the correct
1. Strait of Dover
explanation for Statement-I
2. Strait of Bosphorus
(b) Both Statement-I and Statement-II are
3. Strait of Hormuz
correct and Statement-II is not the correct
4. Cook Strait
explanation for Statement-I
(c) Statement-I is correct but Statement-II is Select the correct answer using the code given

incorrect below.

(d) Statement-I is incorrect but Statement-II is (a) 1-2-3-4

correct (b) 2-1-3-4


(c) 4-1-2-3
95. Consider the following statements with (d) 1-3-4-2
reference to fatty acids:
1. Saturated fatty acids, particularly omega-3 98. Consider the following statements with respect
fatty acids, are considered good fats due to to the gold industry in India:
their beneficial effects on heart health.
1. India is the largest consumer of gold in the
2. Unsaturated fats are exclusively found in
world and accounts for around one-fourth
animal-based foods such as meat and dairy
of global gold demand.
products.
2. India imports more than 80 percent of its
3. Transfats decrease the tendency of blood
gold demand.
to clot.
Which of the statements given above is/are
How many of the above statements are
correct?
correct?
(a) 1 only
(a) Only one
(b) Only two (b) 2 only

(c) All three (c) Both 1 and 2

(d) None (d) Neither 1 nor 2

www.visionias.in 21 ©Vision IAS

Telegram: UPSC_SARTHI
99. Consider the following statements with
reference to Electrified Flex fuel vehicle
(FFV):
1. FFVs have an internal combustion engine
that can run on multiple types of fuel or
fuel mixtures.
2. FFV has both a flex-fuel engine as well as
an electric powertrain.
3. It has higher mileage compared to
traditional gasoline vehicles.
How many of the above statements are
correct?
(a) Only one
(b) Only two
(c) All three
(d) None

100. The terms ‘Attitude Hold’, ‘Terminal


Descent’, ‘Rough Braking’, recently seen in
news, are related to:
(a) Psychoanalysis tools used to diagnose
depression
(b) Phases of meteorite impact on earth’s
surface during past mass extinctions.
(c) Phases of Chandrayaan-3 lander that are
critical for soft landing.
(d) Israel’s all weather air defence system.

Copyright © by Vision IAS


All rights are reserved. No part of this document may be reproduced, stored in a retrieval system or transmitted in
any form or by any means, electronic, mechanical, photocopying, recording or otherwise, without prior
permission of Vision IAS.

www.visionias.in 22 ©Vision IAS

Telegram: UPSC_SARTHI
VISIONIAS
www.visionias.in
ANSWERS & EXPLANATION
GENERAL STUDIES (P) 2023 – ABHYAAS TEST – 4359

Q 1.B
• ETFs are a sort of investment fund that combines the best features of two popular assets: They combine
the diversification benefits of mutual funds with the simplicity with which equities may be exchanged.
• An exchange-traded fund (ETF) is a collection of investments such as equities or bonds. ETFs will let one
invest in a large number of securities at once. Investing in an ETF offers numerous advantages. Here are
some of the primary benefits:
o Diversification: Instantly gaining exposure to a diversified set of underlying assets is a key benefit of
a single ETF. Achieving such diversification through the individual purchase of stocks or bonds can
be both challenging and costly. This feature helps mitigate overall portfolio risk by distributing
investments across various sectors and industries.
o Cost efficiency: The expense ratio measures how much of a fund's assets are used for
administrative and other operating expenses. Most ETFs generally boast lower expense ratios
compared to actively managed mutual funds since they passively track an index instead of
maintaining a team of analysts for selecting individual stocks. This results in enhanced long-term
returns for investors. Hence statement 1 is correct.
o Transparency: The majority of ETFs disclose their holdings daily, providing complete transparency
regarding the components of your investment.
o Liquidity: These funds are actively traded on stock exchanges throughout the day, similar to
individual stocks. This allows for convenient buying and selling of shares at prevailing market
prices, offering increased flexibility compared to mutual funds, where buy and sell orders are
executed only at the close of the trading day. Hence statement 2 is correct.
o Tax efficiency: ETFs typically present tax benefits owing to their in-kind creation and
redemption process. This mechanism has the potential to reduce capital gains distributions in
comparison to traditional mutual funds, ultimately optimizing your returns.
o Minimal investment: Numerous ETFs feature comparatively modest minimum investment
requirements, enabling accessibility for investors with smaller capital bases. This flexibility allows
you to initiate the gradual construction of your portfolio and engage in market growth, even with
limited funds.
• Similar to any investment, ETFs carry potential downsides.
o Short-term price fluctuations: The values of ETFs can vary during the day in response to market
shifts, potentially resulting in increased short-term volatility compared to more stable investments.
While a normal mutual fund's value chnages only at the end of the day. Hence statement 3 is not
correct.
✓ So, unlike mutual funds, prices for ETFs fluctuate continuously throughout the day.
o Tracking discrepancy: Certain ETFs may not precisely mirror their underlying index, leading to a
slight variance in performance.

Q 2.D
• Payment and Settlement Systems Act, 2007 (PSS Act, 2007) provides for the regulation and supervision
of payment systems in India and designates the Reserve Bank of India (Reserve Bank) as the authority for
that purpose and all related matters.
• The Act empowers the Reserve Bank of India (RBI) to designate payment systems in India. A
payment system is a system which enables a payment between a payer and a beneficiary, including
clearing, payment, or settlement service, but it does not include a stock exchange. Hence statement 1 is
not correct.
1 www.visionias.in ©Vision IAS

Telegram: UPSC_SARTHI
• The PSS Act 2007 does not prohibit foreign entities from operating a payment system in India and the Act
does not discriminate/differentiate between foreign entities and domestic entities. Hence statement 2 is
not correct.
• The RBI has the authority to regulate and oversee payment and settlement systems to ensure their
efficiency, integrity, and stability. RBI is empowered to initiate a criminal proceeding against the
offender. RBI can even impose fines on the person for contravening certain provisions of the Act.

Q 3.B
• Financial inclusion (FI) means that individuals and businesses have access to useful and affordable
financial products and services that meet their needs – transactions, payments, savings, credit, and
insurance – delivered in a responsible and sustainable way.
• Priority Sectors Lending (PSL) is the role exercised by the RBI to banks, imploring them to
dedicate funds for specific sectors of the economy like agriculture and allied activities, education and
housing, and food for the poorer population. Thus, the inclusion of more sectors under PSL norms will
promote Financial inclusion. Hence, option 1 is correct.
• KYC means Know Your Customer and sometimes Know Your Client. KYC or KYC check is the
mandatory process of identifying and verifying the client's identity when opening an account and
periodically over time. Stringent KYC norms may increase the formalities and documentation for
financial activities and can pose a hurdle for people using financial activities. Hence, option 2 is not
correct.
• Financial literacy will make people aware of different financial instruments and thus it will promote
financial inclusion. Hence, option 3 is correct.
• Closure of banks in economically unviable areas will make banking facilities inaccessible for the
people living in that area and hence, it may exclude people from banking and financial activities.
Hence, option 4 is not correct.
• Hence option (b) is the correct answer.’

Q 4.B
• The Household Consumption Expenditure Survey (HCES) is designed to gauge household spending
habits, providing crucial insights into consumption patterns, living standards, and overall well-being. The
recently released 2022-23 survey conducted by the National Sample Survey Office (NSSO) sheds
light on significant trends and disparities in household spending across rural and urban
India. NSSO has been conducting household consumption expenditure surveys once every five years
since 1972-73. Hence statement 1 is not correct.
• India’s per capita monthly consumption expenditure witnessed a notable increase of 33-40% in
2022-23 compared to 2011-12, reflecting an average annual growth of 3.1% for rural areas and 2.7% for
urban areas.The average monthly per capita consumption expenditure (MPCE) in Indian households
rose by 33.5% since 2011-12 in urban households to ₹3,510, with rural India’s MPCE seeing a
40.42% increase over the same period to hit ₹2,008. Hence statement 2 is correct.
• Among the States, the MPCE is the highest in Sikkim for both rural (₹7,731) and urban areas (₹12,105).It
is the lowest in Chhattisgarh, where it was ₹2,466 for rural households and ₹4,483 for urban household
members.
• Rural-urban disparity in average MPCE was the highest in Meghalaya, followed by Chhattisgarh.
• Estimates of MPCE were based on Modified Mixed Reference Period (MMRP), capturing
expenditure on essential items over specific reference periods. MMRP considered expenditure on specific
items over the “last 7 days” for certain categories and the “last 365 days” for others.
• Adjusting for imputed values of items received free through social welfare programs and other sources,
the average estimated MPCE in 2022-23 was Rs 3,860 in rural and Rs 6,521 in urban India. Considering
imputed values of items received free through social welfare programs, the average estimated MPCE in
2022-23 was Rs 3,860 in rural and Rs 6,521 in urban India.
• Share of expenditure on food: In 2022-23, the share of expenditure on food in rural India was 46 per
cent (Rs 1,750), and in urban India was 39 per cent (Rs 2,530). In 2011-12, it was 52.90 per cent in
rural India and 42.62 per cent in urban India. This has implications for consumer price index-based
inflation. Hence statement 3 is correct.
• The survey highlights a narrowing gap in per-capita spending between rural and urban India over the last
11 years. Shifts in spending patterns reveal a decrease in the proportion of spending on cereals,
driven by rising incomes and subsidized foodgrain schemes. Increased spending on milk, milk
2 www.visionias.in ©Vision IAS

Telegram: UPSC_SARTHI
products, eggs, fish, and meat reflects evolving dietary preferences, with a notable rise in spending on
processed food and dining out.
• The survey provides critical insights for policymakers to fine-tune social schemes and understand
evolving consumer behavior. The HCES is crucial for policymakers to fine-tune social schemes and
understand the impact of policy interventions on healthcare, education, and living standards. For
industries, the survey provides insights into changing consumer behavior, helping them adapt strategies
and tap into emerging markets.

Q 5.C
• In 2022, the Union Ministry of Railways flagged off country’s first freight train with an all -
Aluminium wagon rake. The aluminium wagon rakes are developed to modernise freight transportation
and reduce carbon footprint of Indian Railways.
• Significance of using aluminum wagon rakes:
• Aluminium is 100% recyclable and resistant to corrosion, reducing maintenance costs. Hence the
statement 1 is correct.
• It can reduce import of Nickel and Cadmium consumed by the iron and steel industry for manufacturing
for conventional rakes. It also boost domestic aluminum industry. So it has a potential to reduce Current
Account Deficit ( CAD) for India. Hence the statement 3 is correct.
• Lower carbon footprint if 15-20% weight shift to aluminium wagons, it can reduce 25 Lakh tonnes of
CO2. Hence the statement 4 is correct.
• It also help in making India Railways' logistic more efficient as it can can carry 180 tonnes extra payload
per trip over conventional rakes. Hence the statement 5 is correct.
• Aluminium trains take less time to manufacture and thus can help in speeding up production.
• However, the cost of production is 35% higher as the superstructure is all aluminium. The cost of
maintenance is lower due to higher corrosion and abrasion resistance. Hence the statement 2 is not
correct.
Q 6.A
• Recent context: India's GDP spiked to 8.4% in Q3 FY24, while GVA growth stood at 6.5% during
the same period.
• GDP (Gross Domestic Product) and GVA (Gross Value Added) are both economic indicators used
to measure the performance of a country's economy, but they have different focuses and
interpretations.
• GDP represents the total market value of all goods and services produced within the borders of a
country within a specific period (usually a year). It includes the value of goods and services produced by
all sectors, including government, private businesses, and households.
• GDP is calculated from the expenditure method using the formula: GDP = Consumption +
Investment + Government Spending + Net Exports (Exports - Imports).
• GVA represents the contribution of each sector (such as agriculture, manufacturing, and services)
to the economy. It is the value of output minus the value of intermediate consumption. GVA focuses on
the value added at each stage of production, excluding intermediate goods and services to avoid
double-counting.
• GVA is calculated using the formula: GVA = Output - Intermediate Consumption.
• GVA (Gross Value Added) can be used to calculate GDP (Gross Domestic Product) by adding the taxes
on products and subtracting the subsidies. The relationship between GVA and GDP can be expressed
through the following formula:
o GDP = GVA + Taxes on Products − Subsidies on Products
o Thus, GDP can increase even while GVA is constant. If the increase in taxes on products
exceeds the decrease in subsidies, it can lead to a situation where GDP increases even if GVA
remains the same.
• In Q3FY24, while India’s GDP growth surged to 8.4%, the GVA growth moderated to 6.5%, which was
in line with expectations but lower than the GDP figures.
• This discrepancy or divergence was attributed to a significant jump in net indirect tax growth and
reduced fertilizer subsidy disbursements. The gap between GDP and GVA, which had averaged
around 20 basis points (bps) in the last eight quarters, widened to 190 bps during this period.
Hence, both statements I and II are correct and statement II is the correct explanation for
statement 1.
• Hence, option (a) is the correct answer.
3 www.visionias.in ©Vision IAS

Telegram: UPSC_SARTHI
Q 7.C
• In the startup world, the terms unicorns, gazelles, and cheetahs are colloquial terms where a unicorn is a
startup company with a value of over $1 billion. A cheetah is a startup that could be a unicorn in four
years, and a gazelle could be one in two years.
• Gazelle are start-ups funded in the 2000s, having worth over $500 million, is not listed yet, and is most
likely to become a unicorn within the next two years.
• Cheetahs are start-ups formed after the year 2000, having a valuation between $200-500 million, and
have the potential to become a Unicorn in the next five years.
o Decacorn: a current valuation of over USD 10 billion.
o Unicorns: Start-ups founded after the year 2000 with a valuation of USD 1 billion.
• India has emerged as the 3rd largest ecosystem for startups globally as of 31st May 2023. India ranks 2nd
in innovation quality with top positions in the quality of scientific publications and the quality of its
universities among middle-income economies.
• Hence, option (c) is the correct answer.

Q 8.A
• Five-Year Plans (FYPs) are centralized and integrated national economic programs. Joseph Stalin
implemented the first FYP in the Soviet Union in the late 1920s.
• Though the planned economic development in India began in 1951 with theinception of First Five Year
Plan , theoretical efforts had begun much earlier , even prior to the independence. Setting up of National
Planning Committee by Indian National Congress in 1938 , The Bombay Plan & Gandhian Plan in 1944,
Peoples Plan in 1945 (by post war reconstruction Committee of Indian Trade Union), Sarvodaya Plan in
1950 by Jaiprakash Narayan were steps in this direction.
• The growth rate target of first FYP was 2.1% but India achieved the growth rate of 3.6%. Therefore, the
first FYP was successful. The growth rate target for second FYP was 4.5% but India could able to achieve
only 4.3% of growth rate. Therefore, the second FYP was not successful. Hence statement 1 is not
correct.
• Eight FYP ( 1992-1997: The plan undertook drastic policy measures to combat the bad economic
situation and to undertake an annual average growth of 5.6% through introduction of fiscal & economic
reforms including liberalisation under the Prime Minister ship of Shri P V Narasimha Rao. It undertook
Indicative planning for the first time( not Ninth FYP) , replacing the earlier centralised planning.
• Eleventh FYP (2007-2012) : Eleventh Plan was aimed “Towards Faster & More Inclusive
Growth". India had emerged as one of the fastest growing economy by the end of the Tenth Plan. The
savings and investment rates had increased , industrial sector had responded well to face competition in
the global economy and foreign investors were keen to invest in India. But the growth was not perceived
as sufficiently inclusive for many groups , specially SCs , STs & minorities as borne out by data on
several dimensions like poverty, malnutrition, mortality, current daily employment etc. Hence statement
2 is not correct.
• A major new initiative taken in the Twelfth Plan, to improve implementation of the Centrally Sponsored
Schemes to rationalise the number of the CSS, reducing them from 142 to 66, and also permitting
greater flexibility in the guidelines. Hence statement 3 is correct.
• Recognising the fact that “one size fits all” national guidelines do not take into account the characteristics
of different States, which justifies a differentiated approach, a new system has been introduced with two
major changes.

Q 9.B
• The Amrit Kaal Vision 2047 Action Plan have been defined across 11 theme which are provided as
below:
o Lead the world in safe sustainable and Green Maritime Sector
o Promote Ocean, Coastal & River Cruise Sector. Hence option 3 is correct.
o Enhance Modal Share of Coastal Shipping & Inland Water Transport
o Promote Maritime Clusters
o Provide Maritime Professional Services
o Become a Global Player in Shipbuilding, Repair & Recycling. Hence option 4 is correct.
o Develop World Class Education, Research & Training Facilities
o Strengthen India’s Global Maritime Presence
o Develop World Class Next Generation Ports
4 www.visionias.in ©Vision IAS

Telegram: UPSC_SARTHI
o Enhance Efficiency Through Technology and Innovation
o Enhance India’s Tonnage
• The Amrit Kaal Vision 2047 Action Plan does not have any element of Naval Power. Hence option 1 is
not correct.
• The Amrit Kaal Vision 2047 does not envisage any deep sea mining mission. Hence option 2 is not
correct.
• Hence option (b) is the correct answer.

Q 10.C
• A Cost Inflation Index table is used to calculate the long term capital gains from a transfer or sale of
capital assets. Capital gain refers to the profit acquired from the sale/transfer of any capital assets,
including land, property, stocks, shares, trademarks, patents, etc.
• The Central Board of Direct Taxes (CBDT) of the Ministry of Finance announces the Cost Inflation
Index for a particular financial year.
• In accounting, usually, long term capital assets are recorded at their cost price in books. Thus, despite
rising prices of assets, these capital assets cannot be revalued. Thus, at the time of sale of these assets, the
profit or gain acquired from them remains high due to their high sale price in comparison to their purchase
price. As a result, assessees also have to pay a higher income tax on the gains from these assets.
• With the application of Cost Inflation Index for capital gain, in the long run, the purchase price of assets is
adjusted according to their sale price, leading to lower profits and lower tax amount on them.
• Hence option (c) is the correct answer.

Q 11.D
• Progressivism is a philosophy that advocates for advancement in science, technology, economic
development, and social organization. It aims to improve the human condition and embraces
change and progress.
o In the context of the Indian Constitution, this principle is reflected in Article 48, which directs the
state to organize agriculture and animal husbandry on modern and scientific lines. This is part of
the broader aim of the Directive Principles of State Policy to establish a welfare state by securing
social and economic justice. Hence option (d) is the correct answer.
o Other articles include which reflects progressivism:
✓ Article 39: This article embodies the principles of progressivism by directing the state to ensure
that the ownership and control of material resources are distributed in a manner that serves the
common good. It also emphasizes preventing the concentration of wealth and means of
production to the detriment of the common interest.
✓ Article 41: This article reflects progressivist ideals by calling upon the state to ensure the right to
work, education, and public assistance in cases of unemployment, old age, sickness, and
disablement. It highlights the state's responsibility to provide social security and welfare measures
to protect the economic interests of vulnerable sections of society.
✓ Article 42: Article 42 underscores the progressivist notion of social justice by directing the state
to make provisions for securing just and humane conditions of work and maternity relief. It
emphasizes the importance of ensuring fair wages, decent working conditions, and maternity
benefits for workers, particularly women.
✓ Article 47: This article embodies progressivist principles by directing the state to regard the
improvement of public health as among its primary duties. It emphasizes the importance of
promoting public health measures, including the provision of medical care, sanitation, and
nutrition, to enhance the well-being of the population.
• Anarchism, cluster of doctrines and attitudes centred on the belief that government is both harmful
and unnecessary. Anarchist thought developed in the West and spread throughout the world, principally
in the early 20th century. Hence option (a) is not correct.
o Derived from the Greek root anarchos meaning “without authority,” anarchism, anarchist, and
anarchy are used to express both approval and disapproval.
• Nihilism is a philosophical belief that denies the existence of inherent meaning or value in life.
Nihilists reject traditional values, beliefs, and moral principles, viewing them as baseless or
meaningless.
o Nihilism's rejection of traditional values and structures would likely be contrary to the notion of
organizing agriculture and animal husbandry based on scientific principles. Hence option (b) is not
correct.
5 www.visionias.in ©Vision IAS

Telegram: UPSC_SARTHI
• Conservatism is a cultural, social, and political philosophy that seeks to promote and to preserve
traditional institutions, customs, and values. It is a political philosophy based on tradition and social
stability, stressing established institutions, and preferring gradual development to abrupt change.
Hence option (c) is not correct.

Q 12.D
• Important powers of Supreme Court
o Article 137 states that the Supreme Court itself is not bound by its own decisions, and the Court
can review its own judgments and orders at any point of time in the future. Hence, statement I is
not correct.
o Article 129 for SC and 215 for High Court states that these courts be considered as the Court of
Records. It means their judicial proceedings are recorded and recognized as legal precedence
and legal references. Hence, statement II is correct.
o Article 141 of the Indian Constitution states that the decisions made by the Supreme Court are binding
on all other courts within the territory of India.
o Article 144 states that all authorities, civil and judicial, in the territory of India shall act in aid of the
Supreme Court.

Q 13.B
• Recent context: The Tamil Nadu governor refused to read out the customary address and walked
out of the House.
• According to Article 176 of the constitution, the Governor has the power to address a sitting of the
legislature. The special power is concerning two occasions. The first is to address the opening session of
a new legislature after a general election. The second is to address the first sitting of the legislature each
year. Commonly referred to as the President’s or Governor’s Address, they are a constitutional
requirement. Hence, statement 1 is correct.
• The Governor’s speech follows the convention of the British system, where it contains legislative and
policy proposals that the government intends to initiate. The speech also recaps the government’s
accomplishments in the previous years.
• The Constitution, under Article 163 respectively, binds the Governor to act on the aid and advice of
the Council of Ministers of the state government on a majority of issues. Therefore, the speech that
the President or the Governor reads before the legislature is the viewpoint of the government and is
prepared by it. The contents of the speech are put together by aggregating inputs from various
ministries of the government. Hence, statement 2 is not correct.
• According to Andul Gafoor Habibullah v. Speaker, West Bengal Assembly (1966) the governor cannot
decline to deliver his address and refuse to fulfil his constitutional duty. Thus, the address under Article
176 is mandatory. However, the HC held that when the governor fails to deliver his address under Article
176 and walks out of the House after laying down the address on the table of the House, this is mere
irregularity, not illegality.
• There is no provision in the constitution mandating the Governor to stick to the portions of speech.
There can be situations when they deviate from the text of the speech prepared by the government.
So far, there have been no instances of the President doing so. But there has been an occasion when
a Governor skipped a portion of the address to the Assembly. Hence, statement 3 is not correct.

Q 14.B
• The Rajamannar Committee was a committee appointed by the Government of India to review the
centre-state fiscal relationship. Hence pair 1 is correctly matched.
o The Rajamannar Committee's report had a significant impact on the centre-state fiscal
relationship. It helped to improve the financial position of the states and to give them greater control
over their own finances. The committee's recommendations also helped to create a more equitable and
sustainable fiscal system.
• The Justice Mulla Committee was a committee appointed by the Government of India to review the
prison system in the country. Hence pair 2 is not correctly matched.
o It provided a comprehensive overview of the problems facing the prison system and offers a number
of practical recommendations for improving it.
• The Justice G. Rohini Commission was constituted by the Government of India to recommend
subcategorization of Other Backward Classes (OBCs). Hence pair 3 is not correctly matched.

6 www.visionias.in ©Vision IAS

Telegram: UPSC_SARTHI
• The Committee's report was based on the following factors:
o The need to provide more equitable representation to OBCs from different social, economic, and
educational backgrounds.
o The need to ensure that OBCs from disadvantaged groups are not left behind.
o The need to take into account the regional variations in the social and economic status of OBCs.
• The Ranganath Mishra Commission is the name of the National Commission for Religious and
Linguistic Minorities that was constituted by the Government of India. Hence pair 4 is correctly
matched.
• Some of the key recommendations of the Ranganath Mishra Commission include:
o Providing constitutional status to the National Commission for Religious and Linguistic Minorities.
o Implementing affirmative action measures for minorities in education, employment, and other areas.
o Establishing special courts to deal with crimes against minorities.
o Promoting communal harmony and understanding between different communities.

Q 15.C
• Furlough and parole envisage a short-term temporary release from custody; While parole is
granted for the prisoner to meet a specific exigency, furlough may be granted after a stipulated
number of years have been served without any reason.
• The grant of furlough is to break the monotony of imprisonment and to enable the convict to maintain
continuity with family life and integration with society.
• Parole and Furlough are the rights which are available to the prisoners under Section - 432 of Criminal
Procedure Code 1973 which is the power to suspend or remit sentences. The Prisoners Act of 1894 and
Prisoners Act, 1900 defines the furlough and the parole system.
• Key differences
o Both parole and furlough are conditional release. Although furlough can be claimed without a reason,
the prisoner does not have an absolute legal right to claim it.
o Parole can be granted in case of short-term imprisonment whereas in furlough it is granted in case of
long-term imprisonment.
o Duration of parole extends to one month whereas in the case of furlough it extends to fourteen days
maximum.
o Parole is granted by Divisional Commissioner and furlough is granted by the Deputy Inspector
General of Prisons.
o For parole, specific reason is required, whereas furlough is meant for breaking the monotony of
imprisonment.
o The term of imprisonment is not included in the computation of the term of parole, whereas it is vice
versa in furlough. Period of furlough is treated as a period spent in the prison. But period spent on
parole is not to be counted as remission of sentence.
o Parole can be granted number of times whereas there is limitation in the case of furlough.
o Since furlough is not granted for any particular reason, it can be denied in the interest of the society.
• Under Article 72 of the Constitution, the President shall have the power to grant pardons, reprieves,
respites or remissions of punishment or to suspend, remit or commute the sentence.
• Respite: It denotes awarding a lesser sentence in place of one originally awarded due to some special
fact, such as the physical disability of a convict or the pregnancy of a woman offender.
• Reprieve: It implies a stay of the execution of a sentence (especially that of death) for a temporary
period. Its purpose is to enable the convict to have time to seek pardon or commutation from the
President.
• Hence option (c) is the correct answer.

Q 16.D
• Recent context: 78 Opposition MPs were suspended, the most ever in a day during the 2023 winter
session of parliament.
• According to the Rule 256 and Rule 374 of Rajya Sabha and Lok Sabha respectively - "Naming a
Member" refers to the drawing of attention of the House by the Chairman/Speaker to the conduct
of a member who disregards the authority of the Chair or abuses the Rules of the House by
persistently and willfully obstructing the business thereof, with a view to action being taken to
suspend him from the service of the House for a period not exceeding the remainder of the session.

7 www.visionias.in ©Vision IAS

Telegram: UPSC_SARTHI
• The consequences of suspension include:
o The maximum period of suspension is for the remainder of the session.
o Suspended members cannot enter the chamber or attend the meetings of the committees.
o He will not be eligible to give notice for discussion or submission.
o He loses the right to get a reply to his questions.
• Hence, option (d) is the correct answer.

Q 17.D
• Differences in powers of Lok Sabha and Rajya Sabha:
o Lok Sabha can introduce and enact Money Bills. But the Rajya Sabha can only suggest
amendments to Money Bills, and can’t initiate, reject, or amend Money Bills.
o Constitutional Amendments can be introduced in either by Lok Sabha or Rajya Sabha.
o MPs from both houses participate in the election and removal of the President, Vice President,
and Judges of the Supreme Court and High Court.
o Rajya Sabha can alone initiate the procedure for the removal of the Vice President. Hence,
option (d) is the correct answer.
o Both houses hold the executive accountable by asking questions and introducing motions and
resolutions.
o In case of any matter related to States, it must be referred to the Rajya Sabha, its approval is
necessary.

Q 18.C
• The Union Public Service Commission (UPSC) is the central agency responsible for conducting the civil
services examinations in India. It was established under Article 315 of the Constitution of India.
• UPSC is the authorised body to release the appointment norms for State DGP in India because it is
the central agency responsible for conducting all civil services examinations in India. The Commission is
responsible for ensuring that the appointment of State DGPs is made fairly and transparently.
Hence, statement 1 is correct.
• UPSC has issued new guidelines for the appointment of DGPs in the country. According to the new
guidelines, only police officers with at least six months of service left before retirement will be
considered for appointment as DGP.Hence, statement 2 is correct.
• UPSC has also reduced the minimum experience required to be eligible for the post of DGP from 30
years to 25 years. Hence, statement 3 is correct.
o The UPSC has taken this decision to ensure that the DGPs have enough time to serve in the post and
to provide stability to the police force. The new guidelines will come into effect immediately. The
move is also expected to help in the faster filling up of vacant DGP posts across the country.

Q 19.C
• The Constitution (One Hundred and Twenty-Eighth Amendment) Bill, 2023 was introduced in Lok
Sabha on September 19, 2023. The Bill seeks to reserve one-third of the total number of seats in Lok
Sabha and state legislative assemblies for women. The Bill received the assent of the President and was
published in the gazette. It is now known as the Constitution (106th Amendment) Act, 2023.
• Key features of the Act:
o Reservation for women: The Act reserves, as nearly as may be, one-third of all seats for women in
Lok Sabha, state legislative assemblies, and the Legislative Assembly of the National Capital
Territory of Delhi. This will also apply to the seats reserved for SCs and STs in Lok Sabha and
state legislatures. Hence options 1 and 4 are not correct options 2 and 3 are correct.
o Commencement of reservation: The reservation hall come into effect after an exercise of
delimitation is undertaken for this purpose after the relevant figures for the first census taken after
commencement of the Constitution (One Hundred and Sixth Amendment) Act, 2023. Based on the
census, delimitation will be undertaken to reserve seats for women. The reservation will be
provided for a period of 15 years. However, it shall continue till such date as determined by a
law made by Parliament.
o Rotation of seats: Seats reserved for women will be rotated after each delimitation, as determined by
a law made by Parliament.

8 www.visionias.in ©Vision IAS

Telegram: UPSC_SARTHI
Q 20.A
• The President gave her assent to the Chief Election Commissioner and other Election Commissioners
(Appointment, Conditions of Service and Term of Office) Bill, 2023. The Act replaces the Election
Commission (Conditions of Service of Election Commissioners and Transaction of Business) Act,
1991. It aims to regulate the appointment, conditions of service, and term of office of the Chief
Election Commissioner (CEC) and other Election Commissioners (ECs), as well as the procedure for the
transaction of business by the Election Commission.
• The CEC and other ECs shall be appointed by the President on the recommendation of a Selection
Committee. Hence statement 1 is not correct.
• Selection Committee consists of the Prime Minister as Chairperson, the Leader of the
Opposition/leader of the largest opposition party in the Lok Sabha Union Cabinet Minister to be
nominated by the Prime Minister. It excludes the office of CJI. Hence statement 2 is not correct.
• The Chief Election Commissioner (CEC) and other Election Commissioners are not eligible for
reappointment, and the removal process for the CEC is similar to that of a Supreme Court judge, while
an Election Commissioner can be removed based on the CEC's recommendation. Hence statement 3
is correct.

Q 21.B
• As per the Census 2011, the tribal population constitutes about 8.6% of the total population in
India.

• Hence option (b) is the correct answer.


9 www.visionias.in ©Vision IAS

Telegram: UPSC_SARTHI
Q 22.D
• The amount and the intensity of insolation vary during the day, in a season, and a year. The factors that
cause these variations in insolation are
o the rotation of the earth on its axis;
o the angle of inclination of the sun’s rays;
o the length of the day;
o the transparency of the atmosphere;
o the configuration of land in terms of its aspect. The second factor that determines the amount of
insolation received is the angle of inclination of the rays. This depends on the latitude of a place. The
higher the latitude the less the angle they make with the surface of the earth resulting in slant sun rays.
The area covered by vertical rays is always less than the slant rays. If more area is covered, the energy
gets distributed and the net energy received per unit area decreases. Moreover, the slant rays are
required to pass through greater depth of the atmosphere resulting in more absorption, scattering, and
diffusion
• The insolation received at the surface varies from about 320 watts/m2 in the tropics to about 70
watts/m2 in the poles. Maximum insolation is received over the subtropical deserts, where the
cloudiness is the least.
• Generally, at the same latitudes the insolation is more over the continent than oceans because clouds
over the ocean reflect back the sunlight.
• In winter, the middle and higher latitudes receive less radiation than in summer. Hence, statement 2 is
not correct.

Q 23.D
• In mountainous areas, the decrease in temperature with increasing altitude leads to the
corresponding change in natural vegetation. As such, there is a succession of natural vegetation belts in
the same order as we see from the tropical to the tundra region.
• The wet temperate type of forests are found between a height of 1000 and 2000 meters. Evergreen
broad-leaf trees, such as oaks and chestnuts predominate. Between 1500 and 3000 meters, temperate
forests containing coniferous trees, like pine, deodar, silver fir, spruce, and cedar, are found. These
forests cover mostly the southern slopes of the Himalayas, places having high altitudes in southern and
northeast India. At higher elevations, temperate grasslands are common.
• At high altitudes, generally, more than 3,600 meters above sea level, temperate forests and
grasslands give way to the Alpine vegetation. Silver fir, junipers, pines, and birches are the common
trees of these forests. However, they get progressively stunted as they approach the snow line. Ultimately,
through shrubs and scrubs, they merge into the Alpine grasslands. These are used extensively for grazing
by nomadic tribes, like the Gujjars and the Bakarwals.
• At higher altitudes, mosses and lichens form part of tundra vegetation. The common animals found in
these forests are Kashmir stag, spotted dear, wild sheep, jack, rabbit, Tibetan antelope, yak, snow leopard,
squirrels, Shaggy horn wild ibex, bear and rare red panda, sheep and goats with thick hair.
• Therefore, the correct sequence of vegetation in the ascending order is Wet temperate Evergreen broad-
leaf forest-Temperate grasslands-Alpine grasslands -Mosses and Lichens.
• Hence option (d) is the correct answer.

Q 24.A
• Context: NASA's Terra satellite observed an impressive example of cavum clouds over the Gulf of
Mexico near Florida’s west coast on January 30, 2024.
• Cavum clouds:
o Scientists have long speculated about Cavum clouds, also called hole-punch clouds or fallstreak holes,
but it's now well understood that these odd cloud formations are caused by airplanes.
Cavum clouds form when planes fly through banks of altocumulus clouds, mid-level clouds
that have supercooled (below the freezing point of water but still liquid) water droplets.
Hence option (a) is the correct answer.
o As air moves around the plane, a process called adiabatic expansion can make the droplets freeze into
ice crystals. The ice crystals eventually grow heavy and fall out of the sky, leaving a hole in the cloud
layer.
o The falling ice crystals that are visible in the center of the holes as wispy trails of precipitation
that never reach the ground - features called virga.
10 www.visionias.in ©Vision IAS

Telegram: UPSC_SARTHI
Q 25.D
• A team of researchers from the Indian Institute of Technology (IIT) Ropar led by Dr Resmi
Sebastian has uncovered the presence of tantalum, a rare metal, in the sands of the Sutlej River in
the Indian state of Punjab. This discovery is crucial given tantalum's widespread application in
electronics and semiconductors. Tantalum is a rare metal with the atomic number 73, known for its grey
colour, heaviness, and exceptional hardness. It is among the most corrosion-resistant metals, forming an
oxide layer when exposed to air, making it resistant to strong and hot acid environments. The rare metal's
highly corrosion-resistant property makes it suitable for use in corrosive environments. This property is
advantageous in electronic components that may be exposed to harsh conditions or reactive
substances. Hence, statement 1 is correct.
• Shale gas & oil is defined as natural gas & oil from shale formations. The shale acts as both the
source and the reservoir for these unconventional hydrocarbons. Older shale wells were vertical while
more recent wells are primarily horizontal and need artificial stimulation, like hydraulic fracturing, to
produce. Only shale formations with certain characteristics will produce gas and oil.ONGC has estimated
187.5 TCF of shale gas in 5 basins (Cambay Onland, Ganga Valley, Assam & Assam-Arakan, Krishna
Godavari Onland & Cauvery Onland). Hence, statement 2 is correct.
• The largest offshore oil reserves in India is at Mumbai High off the coast of Maharashtra. Recently,
Oil and Natural Gas Corporation (ONGC), an Indian state-owned entity, has made two major oil
and gas discoveries in the Arabian Sea blocks off the Mumbai coast. The first find was made in the
725km² Amrit block, while the second was in the 4,668km² Moonga block. Hence, statement 3 is
correct.
• Coal, Oil and natural gas can be obtained from under the seabed. Many minerals are also present in
seawater. The most common one is salt. Seawater is evaporated to get salt. India has large reserves
of beach sand minerals, such as ilmenite, rutile, zircon, monazite, sillimanite and garnet. These
deposits are mostly located in the coastal stretches of peninsular India. Ilmenite is the largest constituent
of the Indian beach sand deposits, followed by sillimanite and garnet. India is one of the leading producers
of garnet in the world. Hence, statement 4 is correct.

Q 26.D
• Indian rock system from a geological perspective can be classified under four categories- the Archaean
Rock System, The Purana rock system, The Dravidian Rock System, and The Aryan Rock System.
• Archaean Rock System: Precambrian rock system. It includes the Archaean Gneisses and Schist (4
billion years old) and Dharwar (1 to 4 billion years old). Archaean Gneisses and Schist System is the
oldest in India. Dharwar System was a highly metamorphosed sedimentary rock system. They are the
oldest metamorphosed rocks. They are economically important because of valuable minerals like iron ore,
manganese, copper, etc.
11 www.visionias.in ©Vision IAS

Telegram: UPSC_SARTHI
• The Purana Rock System (600 to 1400 million years old) is divided into the Cuddapah and Vindhya
systems. The Cuddapah system has unfossiliferous clay, slate, limestone, etc. It is best observed in the
Cuddapah district of Andhra Pradesh. The Vindhya system derives its name from the Vindhya mountain
range. A large area of this system is covered by the Deccan Traps. It is devoid of metalliferous minerals
but provides a large quantity of durable stone, limestone, etc.
• The Dravidian Rock System (600 to 300 million years ago) is found in the extra-Peninsular region. The
rocks of the Carboniferous period fall under the Dravidian Rock System. Coal formation started in the
Carboniferous age.
• The Aryan Rock System is categorized into four parts- the Gondwana system, the Jurassic system, the
Deccan Trap, and the Tertiary system.
• Gondwana rock system formed during the Permian period (around 250 million years ago).

Q 27.B
• From the analysis of various direct and indirect sources like seismic waves, volcanoes, magnetic fields,
etc it is clear that the interior of the earth has a three-layered structure of different densities.
• These three layers are
o Crust,
o Mantle, and
o Core.
• Apart from these three main layers, there are other layers like lithosphere, asthenosphere, and
mesosphere.
• The lithosphere (litho: rock; sphere: layer) is the strong, upper 100 km of the Earth. The lithosphere is the
tectonic plate. The asthenosphere (asthenos: weak) is the weak and easily deformed layer of the Earth that
acts as a “lubricant” for the tectonic plates to slide over. The asthenosphere extends from 100 km depth to
660 km beneath the Earth's surface. Beneath the asthenosphere is the mesosphere, another strong layer.
• The asthenosphere is highly viscous, mechanically weak, and denser than the crust. Hence statement 2 is
not correct.
• The asthenosphere's ductility is much more than either the lithosphere or crust. Hence statement 3 is
correct.
• These properties of the asthenosphere aid in Plate tectonic movement and isostatic adjustment. So, the
Asthenosphere is the main source of magma. Hence statement 1 is correct.

12 www.visionias.in ©Vision IAS

Telegram: UPSC_SARTHI
Q 28.C
• BioTRIG new waste management technology that allows pyrolysis at a community level could help
rural Indians cut indoor air pollution, improve soil health, and generate clean power.
• It works by sealing the waste inside an oxygen-free chamber and heating it above 400 degrees Celsius.
Useful chemicals are produced in the process.
• In the study, the researchers outlined that three products of pyrolysis — bio-oil, syngas and biochar
fertilizer , could help rural Indians live healthier and greener lives.
• Hence option (c) is the correct answer.

Q 29.C
• Eurythermal and stenothermal are terms used to describe an organism's ability to tolerate temperature
changes. Eurythermal organisms can tolerate a wide range of temperatures, while stenothermal organisms
can only tolerate a narrow range of temperatures.
o Eurythermal Animal: Tuna fish are eurythermal and can tolerate a wide range of temperatures, from
cold water near the poles to warm water near the equator.
o Stenothermal Animals: Polar bears are stenothermal animals that can only survive in cold Arctic
temperatures and cannot tolerate warmer temperatures. Hence statement II is not correct.
• Homeostasis is the balanced, internal conditions a living organism maintains. Despite changes in the
environment, all living things have a set of parameters they like to keep constant, such as temperature, salt
concentration, or water balance.
13 www.visionias.in ©Vision IAS

Telegram: UPSC_SARTHI
• Polar bears maintain homeostasis through their body systems as well as specific adaptations for the
environment. Like all animals, polar bears use their body systems to maintain homeostasis. The
respiratory and cardiovascular systems maintain a steady concentration of oxygen in the blood and
remove carbon dioxide. The kidneys work with the nervous system to control blood pressure and maintain
homeostasis of water and salt. Polar bears also have specific adaptations for their environment. For
example, polar bears have a thick layer of fat under their skin and hollow hairs that retain air. This helps
to insulate them from the cold and prevent the loss of heat.
• So, polar bears can’t live in heat. Their specialized adaptations make them successful in cold
weather. Their blubber, thick fur, and physiological makeup work together to keep them warm in
the harsh Arctic conditions. However, these same traits hinder their ability to cope with warmer
temperatures. Hence Statement-I is correct Statement-II is incorrect

Q 30.A
• Use of DDT is banned only for the purpose of Agriculture. In very special circumstances warranting the
use of DDT for plant protection work, the state or central Govt. may purchase it directly from M/s
Hindustan Insecticides Ltd. to be used under expert Governmental supervision.
• The use of DDT for the domestic Public Health Programme is restricted up to 10,000 Metric
Tonnes per annum, except in case of any major outbreak of epidemic. Hindustan Insecticides Ltd
is the sole manufacturer of DDT in the country and may manufacture DDT for export to other countries
for use in vector control for public health purposes. Hence statement 1 is not correct.
• India banned use of Endosulfan after the 2011 Supreme Court Order. The widespread use of Endosulfan
in Kerala by Tea Plantation led the Supreme Court to completely ban use of Endosulfan. Hence
statement 2 is correct.
• India is a signatory to Basel Convention. The adoption of amendment to Basel Convention in 1995 also
known as Basel Ban Amendment led to completely ban exports of hazardous waste from developed
countries to developing countries. The Basel Ban applies to export for any reason including
recycling. Hence statement 3 is not correct.
• Hence option (a) is the correct answer.

Q 31.A
• Recently, the National Green Tribunal has given the Uttar Pradesh government just four weeks for an
update on notifying Dhanauri as a wetland. The Noida International Airport at Jewar might cause some
harm to the nearby wetland area.
• Dhanauri is a natural wetland of great significance and is home to over 217 species of birds.The Dhanauri
wetland harbors 23 species of endangered, critically endangered, and threatened birds, and is one of the
most important habitats for the Sarus crane. It has been recognised as an Important Bird Area by Bird Life
International and so recorded by the BNHS.Hence statement 1 is correct.
• Dhanauri wetlands is not recognised as a wetland under Wetland Rules 2017. Recently a petition in
NGT highlighted non-compliance and non-implementation of rule 7 of the Wetlands (Conservation and
Management) Rules, 2017, under the Environment (Protection) Act, 1986 by the UP State Wetlands
Authority, which failed to notify Dhanauri as a Ramsar wetland. Hence statement 2 is not correct.
• The Dhanauri wetland river does not receive water from Yamuna. It receives water from three or four
sources such as rainwater and runoff from fields that are irrigated. Hence statement 3 is not correct.
• Hence option (a) is the correct answer.

Q 32.D
• Extended Producer Responsibility (EPR) is a policy strategy that charges producers, importers and
brand owners for controlling post-consumer waste. It acknowledges that manufacturers should be
responsible for the waste produced during a product’s lifecycle because they have a considerable
effect on the design, packaging, and material selections of their products.
• In India, numerous laws, norms, and regulations govern EPR compliance. The main laws are:
o Plastic Waste Management Rules (PWMR), 2016
o E-Waste (Management) Rules (EWMR), 2022
o Battery Waste Management Rules, 2022
o Hazardous and Other Wastes (Management and Transboundary Movement) Amendment Rules, 2022
(for Waste Tyre)

14 www.visionias.in ©Vision IAS

Telegram: UPSC_SARTHI
• The Ministry of Environment, Forest, and Climate Change issued the Hazardous and Other Waste
(Management and Transboundary Movement) Second Amendment Rules, 2023 ('Amendment') on 18th
September 2023.
o The Amendment, which will come into effect from 1st April 2024, introduces a new chapter in
the Hazardous and Other Waste (Management and Transboundary Movement) Rules, 2016
('Rules') for the implementation of 'Extended Producers Responsibility' ('EPR') for used oil.
• Plastic waste is a major environmental concern due to its non-biodegradable nature and detrimental
effects on ecosystems. Producers are required by plastic EPR requirements to manage plastic waste
proactively and promote recycling.
• The E-Waste (Management) Rules (EWMR) offer comprehensive instructions for handling electronic
trash (for example, laptops, computers, printers etc) in India properly. Producers are required to create
collection facilities or take part in the development of such facilities. These facilities are in charge of
receiving and directing e-waste to approved recyclers.
• Tyre waste poses unique concerns due to its size and durability. The purpose of EPR tyre
recommendations is to ensure correct handling and disposal of used tyres to avoid environmental dangers.
The ERP and provisions for utilising and managing tyre waste by producers and recyclers have been
included in the 9th Schedule of The Hazardous and Other Wastes Amendment Rules, 2022. EPR
guidelines for batteries focus on ensuring the safe disposal and recycling of used batteries.
• Recently, the Ministry of Environment, Forest and Climate Change (MoEFCC) introduced a
notification on Extended Producer Responsibility (EPR) on waste oil. EPR on waste oil is to improve
the circularity of waste oil management. Waste oil is a contaminant that contains harmful substances that
can pollute freshwater and soil.Waste oil can act as a contaminant as it contains benzene, zinc, cadmium
and other impurities that have the potential to pollute freshwater. EPR on waste oil is applicable to
producers, and bulk generators (like industry, railways, transport companies, power transmission
companies, etc.).
• Hence option (d) is the correct answer.

Q 33.C
• Skunks are mammals in the family Mephitidae. They are known for their ability to spray a liquid with a
strong, unpleasant scent from their anal glands. Because of their lack of speed, skunks do not try to
outrun their attackers. Instead, they rely on their rancid spray for protection. The pungent liquid is a mix
of sulphuric chemicals which they can spray accurately at threats up to 3 meters away, with the foul odor
reaching up to a mile away.
• Bombardier beetles use odor as a defense mechanism. When disturbed they eject a hot noxious
chemical spray from the tip of their abdomen. The Bombardier Beetle stores two separate chemicals in its
body, hydroquinone and hydrogen peroxide. Whenever it feels threatened the chemicals mix with some
special enzymes, heat from the reaction brings the foul-smelling liquid to near 100°C and produces gas
that drives the ejection. The Bombardier Beetle is able to spray up to 4 times its body length and the
damage caused can be fatal to attacking insects and small creatures and is painful to human skin.
• When vultures feel threatened they vomit which puts off most predators. Vulture species like Turkey
vulture can propel their vomit up to 10 feet to ward off predators.
• Hence option (c) is the correct answer.

Q 34.A
• Compensatory Afforestation (CA) refers to afforestation and regeneration activities carried out as a way
of compensating for forest land diverted to non-forest purposes. Here "non-forest purpose" means the
breaking up or clearing of any forest land or a portion thereof.
• As per the Forest (Conservation) Act 1980, whenever forest land is to be diverted for nonforestry
purpose usually the conditions relating to transfer, mutation and declaration as Reserve Forest/
Protected Forest the equivalent non forest land for compensatory afforestation and funds for
raising compensatory afforestation etc are to be imposed. For mining purposes additional conditions
like maintaining a safety zone area, fencing and regeneration etc and for major and medium irrigation
projects, catchment area treatment plans are to be stipulated. Hence option (a) is the correct answer.
• As per the Forest (Conservation) Act 1980, as far as possible, the non-forest land for Compensatory
Afforestation (CA) was to be identified contiguous to or in the proximity of Reserved Forest or Protected
Forest.

15 www.visionias.in ©Vision IAS

Telegram: UPSC_SARTHI
Q 35.B
• CSIR-Centre for Cellular and Molecular Biology (CCMB) is collaborating with research groups across
the world on the Diverse Epigenetic Epidemiology Partnership (DEEP) project. The procedure for the
DEEP project is comprehensive. It aims to generate genomic datasets specifically in underrepresented
populations across African, Asian, and North and South American continents.
• This extensive data collection is coupled with the analysis of DNA methylation data and health-related
measures derived from individuals worldwide. Notably, the significance of this endeavor for India lies in
the profound insights it promises into the genetics of Non-Communicable Diseases (NCDs), including
but not limited to type 1 and 2 diabetes, and chronic pancreatitis. Hence statement 2 is correct.
• At the molecular level, DNA (Deoxyribonucleic Acid) serves as the carrier of genetic information.
Structurally, it takes on the form of a double helix, comprised of two linked strands that wind around each
other, resembling a twisted ladder. The process of DNA methylation, a key aspect of epigenetics,
involves the addition of a methyl group to the DNA molecule, influencing its function. Hence
statement 3 is correct.
• The DEEP project, unfolding over five years, is led by a collaboration of researchers from the
University of Bristol, London, and the CSIR Centre for Cellular and Molecular Biology in India.
The rationale behind this initiative underscores the need for diverse genetic databases, as the project has
primarily gathered data from white European populations, resulting in an underrepresentation of global
communities in health studies. Hence statement 1 is not correct.
• The procedure for the DEEP project is comprehensive. It aims to generate genomic datasets specifically in
underrepresented populations across African, Asian, and North and South American continents.
• Hence option (b) is the correct answer.

Q 36.C
• The five-storied lakes of North America—Superior, Michigan, Huron, Erie, and Ontario—the largest
freshwater ecosystem on Earth, have recorded the lowest level of ice in 2024. Since satellite-based
measurements began in 1973, ice coverage at its maximum winter extent exceeds, on average, 40 percent.
In late February 2024, it stood at only about one-tenth of the average maximum. Hence statement I is
correct.
• According to the National Oceanic & Atmospheric Administration (NOAA), global warming was largely
responsible for the lack of ice this winter since January 2024 the hottest January on record due to El Nino,
the warmer phase of the El Nino-Southern Oscillation. In addition fewer and generally shorter intrusions
of blasts of cold air from the Arctic further results in diminishing ice cover. Hence statement II is not
correct.
• About Great Lake region: The Great Lakes are a dominant physical feature of North America and form
part of the political boundary between the United States and Canada. The system includes five Great
Lakes (Superior, Huron, Michigan, Erie, and Ontario), Lake St. Clair and the connecting channels, along
with many harbors and bays. The Region contains nearly 20 percent of the world’s fresh surface water and
has over 9,000 miles of coastline. Most of North America’s fresh surface water (95%) is in the Great
Lakes

• Hence option (c) is the correct option.


16 www.visionias.in ©Vision IAS

Telegram: UPSC_SARTHI
Q 37.B
• An internal carbon price refers to a monetary value that companies place on greenhouse gas
emissions, which businesses can then factor into investment decisions and business operations to
drive positive change. Companies use internal carbon pricing as a strategic tool to manage climate-
related business risks and prepare for a transition to a low-carbon economy. When a business sets an
internal carbon price, a cost is typically assigned to each ton of carbon emitted so this can be factored into
business and investment decisions, incentivising efficiency and enabling low-carbon innovation. Hence
option (b) is the correct answer.
• There are 3 commonly adopted approaches to internal carbon pricing:
o Shadow pricing: A shadow price refers to a monetary value a company assigns to its greenhouse gas
(GHG) emissions on a per ton basis that is typically used to evaluate the potential cost/benefit of
different emissions reduction strategies. No money is transferred with a shadow price.
o Internal carbon fee or charge: An internal carbon charge is a fee that a company charges itself on a
per-ton basis that can be used to drive decision-making at the margin, reduce costs, or align internal
culture with wider decarbonisation efforts. Internal fees are usually assessed at the business unit level
or based upon specific emissions activities (e.g. electricity demand) and can be pooled for use in
business-unit or corporate-wide sustainability efforts.
o Implicit carbon price: An implicit price is based on how much it costs the organisation to implement
emission reduction projects. This is then applied to where GHG emissions are emitted within the
business. A target may already be in place along with a program of investment. Meanwhile, several
implicit carbon prices may appear within the same organisation. Businesses can strategically use the
implicit price in sustainability communications and allocate costs across the business.

Q 38.C
• Pradhan Mantri Matru Vandana Yojana is a maternity benefit program run by the government of India. It
is a conditional cash transfer scheme for pregnant and lactating women of 19 years of age or above for the
first live birth. The scheme envisages payment of Rs 5000 for first child and Rs 6000 to second child
only if the second child is a girl child. There is no further incentive for the birth of more children. Hence
statement 1 is correct.
• The Scheme envisages providing cash incentive amounting to Rs. 5,000/- in two installments directly
to the Bank/Post Office Account of Pregnant Women and Lactating Mother (PW& LM) in DBT
Mode during pregnancy and lactation.
• Maternity benefit for second children is provided only if the second child is a girl. Through this provision
the scheme discourage pre- birth sex selection. Hence statement 2 is correct.

Q 39.D
• Anganwadi workers are women volunteers providing services under the Integrated Child
Development Services (ICDS) programme started by the Ministry of Women and Child
Development (MWCD) on October 2, 1975, in a bid to tackle malnutrition among all pregnant
women, lactating mothers and children in the age group 0-6 years.
• There are over 25 lakh Anganwadi Workers and helpers in the country. The work done by them includes
providing supplementary nutrition, nutrition and health education, health check-ups, immunisation
and pre-school informal education to children. Hence statement 1 is not correct.
• ASHA workers are empowered with knowledge and a drug kit to provide first-contact healthcare or
community-level curative care for minor ailments such as diarrhea, fevers, care for the normal and sick
newborns, childhood illnesses and first aid. Hence statement 2 is not correct.
• They also act as a depot holder for essential health products that a local community might need at any
point in time.
• This includes Oral Rehydration Therapy (ORS) packet, Iron Folic Acid (IFA) tablets, chloroquine,
Disposable Delivery Kits (DDK), oral pills and condoms.
• ANMs are the crucial component which connects the ASHAs and the communities with the public
health system. Over the years, their role has expanded to address communicable and non-
communicable diseases along with traditional Reproductive and Child Health centric functions.
Hence statement 3 is not correct.

17 www.visionias.in ©Vision IAS

Telegram: UPSC_SARTHI
Q 40.A
• Established in 1973 by the late John and Alice Tyler, the Tyler Prize for Environmental Achievement
is one of the first international premier awards for environmental science, environmental health, and
energy. The 2024 Tyler Prize for environmental achievement will be awarded to Johan Rockstrom for
his groundbreaking contributions and pioneering work to the development of the Planetary Boundaries
framework, the science for defining the safe operating space for humanity on Earth.
• The Emerald Awards showcase organizations, projects, and individuals from across Alberta who
are working to address environmental and climate change issues. These environmental awards are the
only program in Canada to recognize environmental excellence across a diverse range of sectors.
• The Kew International Medal is an award given to individuals who have made a significant contribution
to science and conservation. The award was first established in 1992 by the Board of Trustees of the
Royal Botanic Gardens, Kew.
• Hence option (a) is the correct answer.

Q 41.A
• The Tebhaga movement was a significant agrarian struggle in India, particularly in Bengal, initiated in
September 1946 and continuing until 1950. It was organized by the Bengal Provincial Kishan Sabha,
guided by the Communist Party, to implement the Floud Commission recommendation of Tebhaga,
demanding two-thirds of the harvested crops for sharecroppers. Thus it was not supported by the
Rashtriya Swayamsevak Sangh (RSS). Hence, statement 1 is not correct.
• This involvement of women in the communist movement was expanded to a new level. It saw widespread
autonomous action of the“proletariat and semi-proletariat women”, belonging to dalit and tribal
communities. Through their initiatives, they formed Nari Bahinis or women’s brigades and resisted the
colonial police with whatever weapon they could lay their hands on. Hence, statement 2 is correct.
• Although Northern Bengal districts were worst affected by the Tebhaga agitation, the movement
covered a wider region covering every district in eastern, central and western Bengal. The peasants
carved out their Tebhaga elaka or liberated zones, where they instituted alternative administrations and
arbitration courts.
• The Muslim League ministry, then in power in Bengal, responded by proposing a Bargadar Bill in
January 1947, apparently conceding the sharecroppers' demand; but it was soon dropped because
of opposition from within the Muslim League and from the Congress. Hence, statement 3 is not
correct.

Q 42.D
• A variety of forms of mobilization came to be popularized as the Civil Disobedience Movement
crystallized as a major nationalist form of anti-colonial politics.
o For instance, the formation of volunteer corps, and organization of sankirtan processions to move
around towns and villages to popularise the message of swaraj or prabhat pheris, wherein people
including women and children in the villages and towns went around at dawn singing nationalist
songs, tours by activists and leaders organizing public meetings in the rural and urban areas,
organization of magic lanterns shows and secret circulation of booklets containing nationalist
literature in the villages to spread nationalist ideas amongst people and setting up of underground
Congress ashrams became the familiar modes of nationalist activities.
• Nehru’s arrest in April 1930 for defiance of the salt law evoked huge demonstrations in Madras, Calcutta,
and Karachi. Gandhi’s arrest came on May 4, 1930, when he announced that he would lead a raid on
Dharsana salt works on the west coast.
• Gandhi’s arrest was followed by massive protests in Bombay, Delhi, Calcutta, and Sholapur, where the
response was the fiercest.
• After Gandhi’s arrest, the Congress Working Committee sanctioned three significant measures: (i)
Nonpayment of revenue in Ryotwari areas; (ii) No-chowkidari-tax campaign in Zamindari areas;
and (iii) Violation of forest laws in the central provinces.
o Congress Working Committee did not sanction any Voluntary resignations from Provincial
Legislative Assemblies.
• Under pressure and social boycott, many lower-level government officials including police men resigned
from their services.
• Hence option (d) is the correct answer.

18 www.visionias.in ©Vision IAS

Telegram: UPSC_SARTHI
Q 43.B
• The First Carnatic War was fought between 1740-48. This war was because of the Austrian War of
Succession. A brief description of the war is as follows
o In this war, France and Britain were on opposite sides when France sacked Madras (St. Fort George)
o The Carnatic Nawab was alarmed and sent an army against the French. The Carnatic Army was
defeated.
o In 1748, there was a Treaty of Aix-La-Chapelle which concluded the 1st Carnatic War as well.
• The details of the First Carnatic War:
• Carnatic was the name given by the Europeans to the Coromandel coast and its hinterland. The First
Carnatic War was an extension of the Anglo-French War in Europe which was caused by the Austrian
War of Succession.
• The Cause of the War:
• Although France, conscious of its relatively weaker position in India, did not favour an extension of
hostilities to India, the English navy under Commodore Curtis Bennett seized some French ships to
provoke France. France retaliated by seizing Madras in 1746 with the help of the fleet from Mauritius, the
Isle of France, under Admiral La Bourdonnais, the French governor of Mauritius.
• Battle of St. Thome (in Madras): The First Carnatic War is remembered for the Battle of St. Thome (in
Madras) fought between the French forces and the forces of Anwaruddin, the Nawab of Carnatic, to
whom the English appealed for help. A small French army under Captain Paradise defeated the strong
Indian army under Mahfuz Khan at St. Thome on the banks of the River Adyar. This was an eye-opener
for the Europeans in India: it revealed that even a small disciplined army could easily defeat a much larger
Indian army. Further, this war adequately brought out the importance of naval force in the Anglo-French
conflict in the Deccan.
• Robert Clive (who led the East India Company in the Battle of Plassey) is remembered for his
unbridled and unbounded exploitation of such opportunities, both on his own and the Company’s behalf,
and for his military ingenuity. However, as a lowly Company employee, he had to await his moment. His
opportunity came with the outbreak of the First Carnatic War — in part an extension into the Indian
subcontinent of the European War of the Austrian Succession — in which Britain and France supported
rival Indian factions. In September 1746, French troops captured Madras and Clive was taken prisoner.
However, he escaped south to Fort St. David, the English East India Company post at Cuddalore, where
he enlisted in the Company army and played a notable role in the fort’s defense.
• Hence option (b) is the correct answer.

Q 44.A
• Gopalkrishna Gokhale, often regarded as the political mentor of Mahatma Gandhi, belonged to the
category of Moderates within the Congress. Gokhale’s works: Between 1899 and 1902, he was a
member of the Bombay Legislative Council followed by work at the Imperial Legislative Council from
1902 till his death (1915).
• Contributions of Gokhale:
• At the Imperial legislature, Gokhale played a key role in framing the Morley-Minto reforms of 1909.
• He was associated with the Moderate Group of the Indian National Congress (joined in 1889).
• He became president of INC in 1905 in the Banaras session.
• He established the Servants of India Society in 1905 for the expansion of Indian education.
• He was also associated with the Sarvajanik Sabha journal started by Govind Ranade.
• In 1908, Gokhale founded the Ranade Institute of Economics.
• He started an English weekly newspaper, The Hitavada (The People's paper).
• In his address to the Indian Chamber of Commerce, Prime Minister of India, Shri Narendra Modi stated
"We have to revive the manufacturing sector in West Bengal," PM Modi said this as he was addressing a
session. The saying had become popular earlier in the 20th century when Congress leader Gopal Krishna
Gokhale had said this about Bengal.
• Hence option (a) is the correct answer.

Q 45.A
• The traditional Karuppur Kalamkari Painting done using pens, and brushes made of bamboo sticks, palm
stems, date tree, and coconut tree stems, received the Geographical Indication (GI) tag. The certificate was
issued recently by the Geographical Indications Registry based on an application filed by the Tamil Nadu
Handicrafts Development Corporation (Poompuhar). The Karuppur Kalamkari in Thanjavur region(
19 www.visionias.in ©Vision IAS

Telegram: UPSC_SARTHI
Tamil Nadu) is the traditional dye-painted figurative and patterned clothes, made for temples such
as ceiling cloth, umbrella covers, cylindrical hangings, and chariot covers. Hence pair 1 is correctly
matched.
• Note: Pedana Kalamkari also known as the Machilipatnam style of Kalamkari work which involves
vegetable-dyed block-painting of fabric from Andhra Pradesh also has a GI tag.
• Arunachal Pradesh’s Idu Mishmi tribal group, which resides in the Mishmi Hills of Dibang Valley
and Lower Dibang Valley, weaves handloom items recognized as Idu Mishmi textiles. These fabrics
are distinguished by their elaborate geometric designs, which include rhomboid forms, triangles, angles,
and lines. With striking hues like Teei (black), Shu (bright red), and Low (white), the Idu Mishmi fabrics
are used for a variety of items, including hand purses, shoulder bags, men’s coats, ladies’ shirts,
wraparounds, and more. These handloom products are incredibly resilient, eye-catching, and appropriate
for any time of year. Hence pair 2 is not correctly matched.
• The Lakadong Turmeric from West Jaintia Hills, Meghalaya, one of the world’s finest turmeric
varieties with the highest curcumin content of 7-9% (in comparison to 3% or less in other varieties),
is fast becoming a game changer in the economy of the district. Hence pair 3 is not correctly
matched.

Q 46.B
• Some of the earliest evidence of board games comes from the Indus Valley Civilisation in the form of
archeological finds.
• Chaupar (or pachisi) has an even more interesting timeline. It reached its high point during the
Mughal period (as attested by the giant outdoor game board built by Akbar).
o It is also the game that most traditions ascribe to the downfall of Yudishthira in the Mahabharata.
o Various forms of chaupar have flourished in India. The most modern version of this game is ludo.
• The race game gyan chaupar also has a similar history. It was probably invented by Jain monks as a
didactic game in the early part of the second millennium and later adopted by Hindu and Sufi
traditions throughout the centuries of assimilation during the Bhakti period.
o However, it was reintroduced in India as the British game of snakes and ladders from the twentieth
century onwards.
• Chaturanga was one of the more popular aristocratic games of ancient India. Developing around the
sixth century CE, it started off as a didactic game to teach young princes about the four angas (parts) of
the royal army: the infantry, the elephants, the chariot, and the cavalry.
o The game was invented in India before being introduced to the West Asians, who took it to Europe
from where chess, its most modern version, emerged.
• Hence option (b) is the correct answer.

Q 47.C
• The city of Champaner and the hill of Pavagadh lie 45 kilometres to the northeast of Vadodara and 42
kilometres to the southwest of Godhra in the Panchmahal district.
• In 2004, UNESCO conferred the World Heritage Site tag to the Champaner-Pavagadh site during its 28th
session held in Suzhou, China.
• The entire archeological complex of Champaner and Pavagadh is home to religious structures of
Hindu, Jain, and Muslim communities along with fortresses, fortifications, palaces, agricultural
structures, the water-harvesting installations among others.
• The monuments fully blend the Islamic with other architectural styles, and the city of Champaner is the
only complete and standing pre-Mughal Islamic city in India.
• The Champaner and Pavagadh region is also described in Persian literary sources such as Mirat-i-
Sikandari by Sikandar Bin Muhammad, Ain-i-Akbari by Abul Fazal, and Tabqat-i-Akbari and Tarikhi
Farishta by Muhammad Qasim.
• The region of Champaner and Pavagadh was an important junction connecting Gujarat to Malwa, which
made it an important asset for political reasons, economic benefits, and military aspirations. For any ruler
who wished to conquer Malwa, control over Champaner and Pavagadh was the most essential.
• Following the aspirations of his grandfather and father to capture Pavagadh, Mahmud Shah I, the Sultan
of Gujarat, laid siege to Champaner-Pavagadh between April 1483 and December 1484. The 20-month
siege ended with the defeat of Pitati Rawal Jai Singh, the last Rajput ruler, and the transfer of control over
the coveted region to the sultans of Gujarat.

20 www.visionias.in ©Vision IAS

Telegram: UPSC_SARTHI
o Mahmud Begada established a mint in 1484 here. The coins minted here always carried the
inscription ‘Shahar Mukarram’ meaning ‘the illustrious city’. Hence statement 3 is correct.
o Made of silver and copper, the coins were minted here between 1485 and 1537 by Mahmud Begada
and his successors—Muzaffar II, Bahadur Shah, and Mahmud III. After Humayun attacked the city,
he too minted some silver and copper coins.
o The mint on Pavagadh was one of the four mints of the Gujarat Sultanate. The other three mints were
at Ahmedabad, Ahmednagar, and Junagarh.
• Several commemorative structures such as mausoleums or makbaras are also found in and around the
city, such as the tombs of Sakar Khan and Sikandar Khan. An interesting element of the entire complex
was the many water harvesting installations in the form of lakes, ponds, wells, stepwells, tanks, etc.,
among which the helical stepwell stands out. Hence statement 2 is correct.
• The oldest temple at the Champaner-Pavagadh archeological park is the Lakulisa Temple, dated
around the 10th–11th century and situated on the Mauliya Plateau on Pavagadh Hill.
o The Jain temples on Pavagadh Hill date to around the 13th–14th century. The temples belong to
the Digamber sect of Jainism which was prominent in Gujarat.
o The entire complex of the Champaner-Pavagadh site is home to several majestic and intriguing
mosques. Among all the mosques, the Jami Masjid stands apart. Hence statement 1 is correct.

Q 48.D
• The Iberian style of architecture
o It refers to the architectural style prevalent in the Iberian Peninsula, which includes Spain and
Portugal. While this style is not commonly found in India, there are a few examples where elements
of Iberian architecture can be observed. Hence option (d) is the correct answer.
• Features of Iberian Style of Architecture:
o Use of Red Brick and White Plaster: Iberian architecture often incorporates the use of red brick and
white plaster in construction, creating a distinctive color contrast.
o Intricate Tilework (Azulejos): Decorative ceramic tiles, known as azulejos, are a hallmark of Iberian
architecture. These tiles are used to adorn walls, floors, and facades, often featuring intricate patterns
and designs.
o Courtyards and Gardens: Iberian architecture frequently incorporates courtyards and gardens as
central features of buildings, providing spaces for relaxation and socialization.
o Arched Doorways and Windows: Arched doorways and windows are common architectural
elements in Iberian-style buildings, adding to their visual appeal.
o Whitewashed Facades: Many buildings in the Iberian style are characterized by whitewashed
facades, which help to reflect sunlight and keep interiors cool during hot weather.
• Famous Examples of Iberian Style Architecture in India:
o While pure examples of Iberian-style architecture may be rare in India, there are a few structures that
exhibit influences from this architectural tradition:
o Portuguese Churches in Goa: Goa, a former Portuguese colony, is home to several churches built in
the Iberian style. These churches often feature whitewashed facades, ornate interiors, and decorative
azulejos. Examples: Se Cathedral in Goa, Basilica of Bom Jesus, Castella de Aguada in Mumbai, St.
Paul's Church, Diu.
o Portuguese Forts: Along the coast of India, particularly in regions that were under Portuguese rule,
there are forts built in the Iberian style. These forts typically feature thick walls, bastions, and
watchtowers.

Q 49.B
• India finished their Asian Games 2022 campaign with a record haul of 107 medals - 28 gold, 38 silver and
41 bronze. This surpassed India’s previous record tally achieved at the last edition at Jakarta 2018, where
a 570-strong Indian squad racked up 70 medals - 16 gold, 23 silver, and 31 bronze. China led the overall
Asian Games 2022 medals tally with 201 gold, ahead of Japan (52) and the Republic of Korea (42). Only
four countries won over 100 medals in Hangzhou. Hence statement 1 is correct.

21 www.visionias.in ©Vision IAS

Telegram: UPSC_SARTHI
• Twenty20 cricket has been played at the Asian Games twice before, at Guangzhou 2010 and
Incheon 2014, with Bangladesh and then Sri Lanka crowned men’s winners. Hence Statment 2 is
not correct.
• Shooting led the way for India at the Asian Games 2022 with an unprecedented haul of 22 medals,
which included seven gold. Hence Statment 3 is correct.

Q 50.B
• Maulana Abul Kalam Azad Trophy (MAKA): The oldest National Sports Award in India, the Maulana
Abul Kalam Azad Trophy or the MAKA Trophy was instituted in 1956–1957. This award is named in
honour of the Indian freedom fighter and the country’s first education minister Shri Abul Kalam Azad. It
is given to an institution or university for top performance in inter-university tournaments over the
last one year. Hence statment 1 is correct.
• Arjuna Award: This award is named after Arjuna, the main protagonist from the ancient Indian epic
Mahabharata. The Arjuna Award was instituted in 1961. It was India’s highest sporting honour before the
Khel Ratna came into being. It is awarded for consistently good performance over a period of four
years. Hence statement 2 is correct. The winners of the Arjuna Award receive a statuette of Arjuna, a
certificate, and a cash prize. The first Arjuna Award was conferred upon India’s football Olympian PK
Banerjee who was among the 20 recipients of the Arjuna Award in its inaugural edition in 1961.
• Hockey player Anna Lumsden was the first woman Arjuna Award winner, Hence statement is 3 not
correct.
• Olympic bronze medal-winning weightlifter Karnam Malleswari was the first Indian woman to
receive Major Dhyan Chand Khel Ratna Award in 1994-95.

Q 51.A
• The "New Look Policy" refers to a foreign policy approach pursued by the administration of U.S.
President Dwight D. Eisenhower during the 1950s.
o It was primarily formulated by Eisenhower's Secretary of State, John Foster Dulles, and aimed to
address the challenges posed by the Soviet Union and communism during the Cold War era.
• The ‘Act East Policy’ announced in November, 2014 is the upgrade of the “Look East Policy”.
o It is a diplomatic initiative to promote economic, strategic and cultural relations with the vast
Asia-Pacific region at different levels.
o It involves intensive and continuous engagement with Southeast Asian countries in the field of
connectivity, trade, culture, defense, and people-to-people-contact at bilateral, regional, and
multilateral levels.
• India’s Look East policy was enunciated by Prime Minister Narasimha Rao in 1992. Though a large
part of Asia and all of Australasia and the Pacific islands lie to the east, the Look East policy was
conceived initially with reference to South East Asia and started with India becoming a sectoral
dialogue partner of ASEAN, the Association of South East Asia Nations. The policy was later
expanded to include East Asia and Oceania.
• The concept of the Neighbourhood First Policy came into being in 2008. But 'Neighbourhood First'
policy as a foreign policy initiative introduced by India in 2014 under PM Modi government.
o It was conceived to bolster relations with certain priority countries such as Afghanistan,
Bangladesh, Maldives, Myanmar, Nepal, Pakistan and Sri Lanka.
• Hence option (a) is the correct answer.

22 www.visionias.in ©Vision IAS

Telegram: UPSC_SARTHI
Q 52.C
• Voice of Global South Summit has been India's endeavour to provide a common platform to
deliberate on the concerns, interests and priorities that affect the developing countries. During
India’s G20 presidency, India shared the views received from the Voice of Global South Summit.
• India hosted a special virtual Summit, called the Voice of Global South Summit under the theme –
‘Unity of voice, Unity of purpose’ from January 12-13, 2023.
o India will work to ensure that the valuable inputs generated from partner countries in the Voice of
Global South Summit deliberations receive due cognizance globally.
o India's ongoing presidency of the G20 provides India a special and strong opportunity to
channelize these inputs into the deliberation and discourse of the G20.
o It was a new and unique initiative that envisaged bringing together countries of the Global South and
sharing their perspectives and priorities on a common platform across a whole range of issues.
• The Second Edition of the Voice of Global South Summit was held on 17 November 2023
virtually. At the invitation of the Government of India, the Chief Advisor of the Interim Government,
Chogyal Dago Rigdzin represented Bhutan at the Inaugural Leaders’ Session which was chaired by Prime
Minister Narendra Modi.
• The overarching theme of the Summit is “Together for Everyone’s Growth with Everyone’s Trust”. The
initiative was inspired by Prime Minister Shri Narendra Modi's vision of ‘Sabka Saath Sabka Vikas Sabka
Vishwas aur Sabka Prayas’, and also underpinned by India's philosophy of Vasudhaiva Kutumbakam.
• Hence option (c) is the correct answer.

Q 53.C
• Amid Israel’s ongoing military attacks on the Gaza Strip, particularly in its southern region, United
Nations Secretary-General Antonio Guterres has invoked Article 99 of the UN Charter in a bid to
establish a ceasefire.
• The UN Charter is the founding document of the United Nations. Based on the powers conferred through
it, the UN can take action on a wide variety of issues. The Charter is considered an international treaty,
meaning UN Member States are “bound by it”.
• Article 99 states the Secretary-General may bring to the attention of the Security Council any matter
which in his opinion may threaten the maintenance of international peace and security.
• To be adopted, a resolution needs at least nine votes in favor and no vetoes by the five
permanent members – the United States, Russia, China, France or Britain. Hence statement 1 is
correct.
• Article 99 does not immediately result in direct action from the UN or the Security Council, it could
present the Security Council with an impetus to revisit previously failed draft resolutions. Hence
statement 2 is correct.
• The provision has been rarely invoked. Past examples include the upheaval in the Republic of the
Congo in 1960 following the end of Belgium’s colonial rule and a complaint by Tunisia in 1961 against
France’s naval and air forces launching an attack.

Q 54.C
• The Exclusive Economic Zone extends upto 200 nautical miles from the baselines from which the breadth
of the territorial sea is measured.

23 www.visionias.in ©Vision IAS

Telegram: UPSC_SARTHI
• As per article 56 of the United Nations Conference on the Law of the Sea (UNCLOS) rights of coastal
state in the Exclusive Economic Zone (EEZ) are-
o sovereign rights for the purpose of exploring and exploiting, conserving and managing the natural
resources, whether living or non-living. Hence statement 1 is correct.
o sovereign rights with regard to other activities for the economic exploitation and exploration of the
zone, such as the production of energy from the water, currents and winds. Hence statement 2 is
correct.
• The jurisdictions of coastal state in EEZ are:
o the establishment and use of artificial islands, installations and structures; Hence statement 3 is not
correct
o marine scientific research; Hence statement 4 is correct
o the protection and preservation of the marine environment;
• The coastal state doesn't have right to prohibit or limit freedom of navigation or overflight, however some
exception are there as well.

Q 55.C
• Union Home Minister inaugurated the National Automated Fingerprint Identification System, or
NAFIS, which is designed to help in the quick and easy disposal of cases with the help of a
centralized fingerprint database. The identification of the deceased led to the registration of a homicide
case based on circumstantial clues. NAFIS is the Centre’s ambitious scheme under which workstations
were set up in every state for identification through fingerprint and palm print database and
matching system.
• Its software serves as the key application for fingerprint experts for crime and criminal investigation
functions. It also functions as the central information repository of all the crime and criminal-related
fingerprint data of the country. The national-level NAFIS solution is implemented and managed by the
Central Fingerprint Bureau (CFPB) at the National Crime Records Bureau. Hence statement 1 is
correct.
• National Automated Fingerprint Identification System assigns a unique 10-digit National Fingerprint
Number for each criminal, based on biometrics. Hence statement 2 is correct.
• This 10-digit ID will be used for an individual’s lifetime, and different crimes registered under
different FIRs will be linked to the same National Fingerprint Number. The unique ID will be used
for the lifetime of an offender. Different crimes registered under different FIRs will be logged as
incidents belonging to the same National Fingerprint Number. The first two digits of the ID will be the
state code of the state where the criminal is registered, followed by a sequence number. The state partition
will have IDs belonging to a state. Hence statement 3 is correct.
• Madhya Pradesh became the first state in the country to identify a deceased individual through the
National Automated Fingerprint Identification System. Hence statement 4 is not correct.

Q 56.A
• E Ink displays technology is a brand of a special type of screen technology often used in e-readers
like the Amazon Kindle. The technology was originally developed in the 1990s at MIT and is now
owned by E Ink Corporation. E Ink first hit the scene in the early 2000s and was all the rage among e-
readers like the Amazon Kindle, Kobo eReader, and Sony Reader.
• The screens work using tiny microcapsules filled with positively charged white particles and
negatively charged black ones suspended in the fluid inside the display. By applying positive or
negative electrical charges to different areas of the screen, the white or black particles can be made
to rise to the surface, creating the text and images on the display. Hence, statement 1 is correct.
• E Ink and E-Paper may sound like the same thing, but they refer to different display technologies. E-Paper
is a broad term for any display designed to mimic the appearance of real paper. Meanwhile, E Ink is a
specific type of E-Paper display that uses millions of tiny microcapsules filled with positively charged
white particles and negatively charged black particles suspended in a clear fluid.
• Unlike LCD and LED displays that use a backlight, E Ink displays reflect light – just like paper.
This makes them easier on the eyes for long reading sessions. They also require very little power
since they don’t need a backlight and only use energy when the image changes. The lack of
backlighting also means that they are easier to read under brighter lighting conditions, which isn’t
the case with LCD/LED displays at all – legibility takes a hit under bright sunlight. Hence,
statement 2 is not correct.
24 www.visionias.in ©Vision IAS

Telegram: UPSC_SARTHI
• E Ink displays have several key advantages that make them well-suited for certain applications.
o First, they consume very little power compared to LCD and OLED displays.
✓ An E Ink display only draws power when the image is changed, meaning it can display a static
image for weeks or months without needing a charge.
✓ This makes it ideal for e-readers and other devices meant for long battery life.
o Second, E Ink displays cause less eye strain for the user.
✓ The lack of a backlight and the paper-like visual experience means users can read on an E Ink
device for hours without fatigue.
✓ The matte surface and high contrast also make them easily readable outdoors and under bright
light.
• However, E Ink displays also have some downsides. The biggest is their slow refresh rate compared to
LCD and OLED displays. While improvements have been made, such Ink displays still take much longer
to refresh, making them unsuitable for video or animation.
• E Ink also has limitations on colour and resolution compared to other display technologies. And the
niche nature of E Ink manufacturing means the displays remain expensive, especially in larger sizes.
This limits their use to mostly e-readers and a handful of other products where their particular
advantages outweigh the higher cost.

Q 57.B
• Recent context: A rare metal called Tantalum found in the Sutlej River can change India’s semiconductor
industry.
• Tantalum is a very hard, ductile, lustrous, blue-gray transition metal that is highly corrosion-resistant. It is
part of the refractory metals group, which are widely used as components of strong high-melting-point
alloys.
• Tantalum is a rare metal widely used in the electronics and semiconductor industry because of its
unique characteristics. The capacitors made of Tantalum are capable of storing more amount of
electricity without much leakage than any other capacitors.
• This usage makes the metal ideal for use in portable devices, including smartphones, laptops, and
cameras.
• In a significant development that could change the future of the electronics and semiconductor
industry in India, a team of researchers from the Indian Institute of Technology (IIT) Ropar,
Punjab, has discovered the presence of a rare metal called Tantalum, in the Sutlej River.
• Hence, option (b) is the correct answer.

Q 58.D
• What is end-to-end encryption?
o End-to-end encryption is a communication process that encrypts data being shared between two
devices. It prevents third parties like cloud service providers, internet service providers (ISPs), and
cybercriminals from accessing data while it is being transferred.
o The process of end-to-end encryption uses an algorithm that transforms standard text into an
unreadable format.
o This format can only be unscrambled and read by those with the decryption keys, which are only
stored on endpoints and not with any third parties including companies providing the service.
Hence statement 1 is not correct.
• Where is it used?
o End-to-end encryption is used to secure communications. Some of the popular instant messaging apps
that use it are Signal, WhatsApp, iMessage, and Google Messages.
o However, instant messaging is not the only place where user data is protected using end-to-end
encryption. It is also used to secure passwords, protect stored data, and safeguard data on cloud
storage. Hence statement 2 is not correct.
• What does it mean for users?
o End-to-end encryption ensures that user data is protected from unwarranted parties including service
providers, cloud storage providers, and companies that handle encrypted data.
o The data can only be accessed with access to the device passcode, password, recovery contact, or
recovery key. The technology also makes it harder for service providers to share user information
from their services with authorities.

25 www.visionias.in ©Vision IAS

Telegram: UPSC_SARTHI
o However, end-to-end encryption does not protect metadata, which includes information like
when a file was created, the date when a message was sent, and the endpoints between which
data was shared. Hence statement 3 is not correct.

Q 59.A
• The terms Bluejacking, Bluesnarfing, and Juice jacking, sometimes mentioned in the news, are
related to cybersecurity threats exploiting Bluetooth technology.
• Juice jacking or infecting devices tethered to public charging stations has been around for some time.
U.S. law enforcement agency FBI and the Federal Communications Commission (FCC) have issued
warnings detailing the risks posed by such attacks. This form of attack is used to target devices being
charged at USB charging stations in public spaces. In May 2023, the FBI in a tweet advised users to avoid
using free charging stations in airports, hotels, or shopping centers.
• Bluejacking: This refers to a relatively less harmful cyberattack where an attacker sends unsolicited
messages, contact cards, or pictures to a victim's Bluetooth-enabled device. The primary intent of
bluejacking is annoyance rather than data theft.
• Bluesnarfing: This is a more serious cyberattack that exploits security vulnerabilities in Bluetooth
connections. In bluesnarfing, attackers access sensitive data on the target device, such as photos, contacts,
emails, and passwords, by exploiting weaknesses in the Bluetooth Object Exchange (OBEX) protocol.
• Juice jacking: Juice jacking involves tampering with public USB charging ports to steal data or
install malware on devices connected to them. While Bluejacking and Bluesnarfing specifically
target Bluetooth vulnerabilities, Juice jacking focuses on exploiting the trust users place in public
charging stations.
• Hence option (a) is the correct answer.

Q 60.D
• In the human body, most of the DNA in a genome is neatly packed inside cells with the help of
specific proteins, protecting it from being degraded. However, in a variety of scenarios, some
fragments of DNA are ‘released’ from their containers and are present outside the cell, in body
fluids. These small fragments of nucleic acids are widely known as cell-free DNA (cfDNA).
• Scientists have been aware of such degraded fragments of nucleic acids in body fluids since 1948. But
only in the last two decades or so, since genome sequencing technologies started to become more
accessible, have they really figured out what to do with that knowledge.
• cfDNA can be generated and released from a cell in a number of possible situations, including when a
cell is dying and the nucleic acids become degraded. Since an array of processes modulates the
degradation, the amount, size, and source of the cfDNA can vary across a range as well.
• All six of the listed applications involve cfDNA technology: Minimally invasive screens for disease -
related genetic abnormalities: cfDNA technology is utilized for screening genetic abnormalities,
offering a less invasive method compared to traditional approaches.
• Screening pregnancies for trisomy 21: Non-invasive prenatal testing (NIPT) involves the use of cfDNA
to screen for chromosomal abnormalities in the foetus, including trisomy 21.
• The most common method, widely considered the gold standard in prenatal cfDNA Down syndrome
testing, is chromosome microarray analysis (CMA). CMA can analyze thousands of cfDNA fragments
simultaneously, typically using a microchip containing probes specific to target regions of chromosomes.
• Assessment of allograft injury or rejection: cfDNA technology is employed to assess organ transplant
success and detect donor-derived cfDNA (dd-cfDNA), indicating potential allograft injury or rejection.
• Liquid biopsy: Liquid biopsy, a non-invasive method for detecting and monitoring cancer, involves the
analysis of cfDNA released by tumorous cells into the bloodstream.
• Non-invasive prenatal testing: NIPT, as mentioned earlier, is a specific application of cfDNA
technology for screening chromosomal abnormalities during pregnancy.
• Hence option (d) is the correct answer.

Q 61.D
• The Constitution provides for a three-fold distribution of legislative subjects between the Centre and
the states, viz., List-I (the Union List), List-II (the State List) and List-III (the Concurrent List) in
the Seventh Schedule.

26 www.visionias.in ©Vision IAS

Telegram: UPSC_SARTHI
• The Parliament has exclusive powers to make laws with respect to any of the matters enumerated in
the Union List. This list has at present 98 subjects like defence, banking, foreign affairs, currency, atomic
energy, insurance, communication, inter-state trade and commerce, census, audit and so on.
• The state legislature has “in normal circumstances” exclusive powers to make laws with respect to
any of the matters enumerated in the State List. This has at present 59 subjects like public order,
police, public health and sanitation, pilgrimages, other than pilgrimages to places outside India;
agriculture, prisons, local government, fisheries, markets, theaters, betting and gambling and so on.
• Both, the Parliament and state legislature can make laws with respect to any of the matters
enumerated in the Concurrent List. This list has at present 52 subjects like criminal law and procedure,
civil procedure, marriage and divorce, industrial and labour disputes; bankruptcy and
insolvency; population control and family planning, electricity, labour welfare, economic and social
planning, drugs, newspapers, books and printing press, and others.
• Hence option (d) is the correct answer.

Q 62.B
• Liberty, in the realm of political philosophy, refers to the concept of individual freedom and
autonomy. It encompasses the idea that individuals should have the ability to act and make choices
according to their own will, without undue interference or coercion from external sources. There are
two primary types of liberty: positive liberty and negative liberty.
o Negative Liberty: Negative liberty, also known as "freedom from," emphasizes the absence of
external constraints or interference in one's actions.
✓ It asserts that individuals should be free to act as they wish as long as they do not infringe upon
the rights or liberties of others.
✓ Negative liberty is concerned with protecting individuals from coercion, oppression, or
interference by the state or other individuals or entities.
✓ Examples of negative liberty include freedom of speech, freedom of religion, freedom of
assembly, and freedom from arbitrary arrest or detention.
✓ It is often associated with classical liberal and libertarian political ideologies, which prioritize
limiting the role of government to ensure individual autonomy and minimize interference in
private affairs.
o Positive Liberty: Positive liberty, also known as "freedom to," emphasizes the presence of
opportunities and resources that enable individuals to fulfill their potential and achieve their
goals.
o It focuses on the capacity of individuals to actively participate in shaping their own destinies and the
governance of their society.
o Positive liberty is concerned with overcoming internal and external barriers that may inhibit
individual agency and self-determination.
o Examples of positive liberty include access to education, healthcare, employment opportunities,
and political participation.
o It is often associated with social democratic and progressive political ideologies, which advocate for
government intervention and social programs aimed at promoting equality of opportunity and
enhancing individual capabilities.
• In summary, negative liberty emphasizes freedom from external interference or coercion, while
positive liberty emphasizes the presence of opportunities and resources that enable individuals to
exercise their autonomy and achieve their goals. Both types of liberty are essential aspects of a well-
functioning society, and debates often arise regarding the balance between them and the role of
government in promoting and protecting individual freedoms.
• Hence option (b) is the correct answer.

Q 63.A
• The Standing committees are permanent committees that are constituted by the Parliament to deal with
specific areas of public policy or administration. These committees are constituted at the beginning of
each session of Parliament and continue until the end of the session. Hence statement 1 is correct.
• After a committee completes its study, it publishes its report which is laid in Parliament. These
recommendations are not binding; however, they hold a lot of weight. For example, the Standing
Committee on Health made several recommendations to the National Medical Commission Bill in

27 www.visionias.in ©Vision IAS

Telegram: UPSC_SARTHI
2017. Many of these were incorporated in the recently passed 2019 Bill, including removing the
provision for allowing a bridge course for AYUSH practitioners. Hence statement 2 is not correct.
• There are 24 Departmentally Related Standing Committees covering under their jurisdiction all the
Ministries/ Departments of the Government of India. Each of these Committees consists of 31 Members -
21 from Lok Sabha and 10 from Rajya Sabha to be nominated by the Speaker, Lok Sabha and the
Chairman, Rajya Sabha, respectively. The term of Office of these Committees does not exceed one
year. Hence statement 3 is not correct.

Q 64.C
• The Grassroots Innovation Forum, specifically the "ASEAN India Grassroots Innovation Forum
(AIGIF)," is an annual program aimed at fostering cooperation and strengthening the relationship
between India and the ASEAN (Association of Southeast Asian Nations) countries in the field of
science, technology, and innovation (STI).
• India along with 10 ASEAN Member States (AMS) were represented by 200 participants, at the 4th
edition of the annual ASEAN India Grassroots Innovation Forum (AIGIF) that was launched in
Langkawi, Malaysia on 28th November 2023. The AIGIF is an annual programme focused on fostering
a strengthened relationship between India and AMS on the premise of cooperation in Science, Technology
and Innovation (STI).
• The 1st two forums were organized in Indonesia (2018) and Philippines (2019) respectively while there
was a brief pause of two years owing to the pandemic. The three-day 3rd ASEAN India Grassroots
Innovation forum was organized by the ASEAN Committee on Science, Technology and Innovation
(COSTI) in partnership with the Department of Science and Technology (DST), Government of India and
the National Innovation Foundation (NIF) – India during 20-22 December 2022.
• Hence option (c) is the correct answer.

Q 65.B
• The Indian government has officially announced the implementation of the significant Press and
Registration of Periodicals Act (PRP Act), 2023 and its Rules. The Press and Registration of Periodicals
Act (PRP Act), 2023 has become effective from March 1st, 2024. This act replaces the Press and
Registration of Books Act, 1867 legislation from the colonial – era.
• Key features of the act include:
• Registration of periodicals:
o The act provides for the registration of periodicals, which include any publication containing
public news or comments on public news. Hence options 1 and 4 are correct.
o Periodicals do not include books or scientific and academic journals. Hence options 2 and 3 are
not correct.
• The Act provides that a declaration specifying the printer/ publisher be made to the District
Magistrate (DM).
o The DM sends the declaration to the Press Registrar, who then issues a certificate of registration.
o Making such declaration and authentication by the DM is necessary for the publication of the
newspaper.
• The Act allows the publisher of a periodical to obtain a registration certificate by filing an online
application with the Press Registrar General and specified local authority.
o A person who has been convicted of a terrorist act or unlawful activity, or has acted against the
security of the State will not be allowed to publish a periodical.
• The new Act has introduced an online registration system called the Press Sewa Portal.The new Act
mandates that all applications for registering periodicals must be done exclusively online through the
Press Sewa Portal. This system has replaced the old manual processes, which were complex and involved
multiple steps and approvals.
• The Press Registrar General of India (PRGI), formerly known as the Registrar of Newspapers for
India (RNI), will be responsible for implementing the objectives of the new Act.
o The PRGI would be responsible for maintaining a register of periodicals, setting standards for
periodical titles, verifying circulation numbers, and handling registration changes, suspensions, or
cancellations.
• Hence option (b) is the correct answer.

28 www.visionias.in ©Vision IAS

Telegram: UPSC_SARTHI
Q 66.C
• The rules of procedure of the Indian parliament provide for a mechanism to raise points of order. A
point of order is a question raised by a member of parliament on the procedure being followed in
the House. The Speaker is the final authority on whether a point of order is valid or not.
• Some examples of points of order include:
o A member raising a point of order that the quorum is not present in the House.
o A member raising a point of order that a member is speaking out of turn.
o A member raising a point of order that a member is using unparliamentary language.
• Any member of Parliament can raise a point of order, regardless of their party affiliation. Whether a
member belongs to ruling party or opposition party, It does not matter when the point of order is raised by
him. Hence, statement 1 is correct.
• No debate is allowed on a point of order. Hence, statement 2 is correct. The purpose of a point of
order is to ensure that the rules of procedure are being followed, and debate would only serve to delay the
proceedings. The purpose is to have a quick and clear resolution to an issue concerning the rules of
the proceeding.

Q 67.C
• International Organization for Migration was established in 1951, in response to the large number
of internally displaced persons and war refugees in Europe after the Second World War.
o IOM is the leading intergovernmental organization in the field of migration and works closely with
governmental, intergovernmental, and non-governmental partners.
o It is the principal United Nations agency working in the field of migration. The organization
implements operational assistance programs for migrants, including internally displaced persons,
refugees, and migrant workers.
o It has its headquarters in Geneva, Switzerland.
o India became an IOM Member State on 18 June 2008.
• The International Labour Organisation Headquarters is established in Geneva.
o It was created in 1919, as part of theTreaty of Versailles that ended World War I, to reflect the
belief that universal and lasting peace can be accomplished only if it is based on social justice.
• The seat of the International Court of Justice is at the Peace Palace in The Hague (Netherlands).
o It was established in June 1945 by the Charter of the United Nations and began work in April
1946.
o It is the principal judicial organ of the United Nations.
o Four Indians have been members of the ICJ so far.
• Food and Agriculture Organization (FAO) is a specialised agency of the United Nations that leads
international efforts to defeat hunger and improve nutrition and food security.
o Its goal is to achieve food security for all and make sure that people have regular access to enough
high-quality food to lead active, healthy lives.
o Established in 1945, it has its headquarters in Rome, Italy.
o India is one of the founding members of the FAO.
• Hence option (c) is the correct answer.

Q 68.B
• The Legal Aid Defence Counsel System (LADCS) is a project implemented by the National Legal
Services Authority (NALSA) to provide free legal aid to the poor and marginalized sections of
society. Hence, statement 1 is correct. NALSA has played a significant role in providing free legal aid to
the poor and marginalized sections of society. It has also helped to promote awareness of legal rights
among these sections of society.
• LADCS will provide free legal assistance to the accused in custody and the defendants in criminal
cases (not civil cases). Hence, statement 2 is not correct. The system is designed to ensure that
everyone has access to justice, regardless of their financial situation.
• Lawyers who are engaged within the LADCS are not allowed to take any private cases because they
are required to devote their full time and attention to representing clients who are eligible for legal
aid. Hence, statement 3 is correct.

29 www.visionias.in ©Vision IAS

Telegram: UPSC_SARTHI
Q 69.C
• Reflation refers to a fiscal or monetary policy (or policies) aimed towards stimulating the economy
(i.e., increasing the level of economic activity) and combating deflation. It is used to restore long-run
inflation, reduce the time span of an economic contraction, or reignite an expansionary business cycle.
Reflation may also be used to describe the period immediately following a period of economic
contraction.
• During an economic contraction, there is often deflationary pressure on prices. Deflationary pressures are
caused by lower aggregate demand, which causes firms to reduce their output and/or prices. To re-
stimulate the economy, the government and/or central bank enacts policies aimed at increasing economic
participation.
• Reflation policies that may be enacted include:
o Increasing the money supply, which puts more money into the hands of consumers, thereby creating
more liquidity in the economy. Hence option 1 is correct.
o Lowering the tax rate, which makes firms and employers wealthier with hopes that they will inject
that money into the economy. Hence option 2 is correct.
o Reducing interest rates, which makes the cost of borrowing cheaper, thereby creating more liquidity
in the economy. Hence option 3 is not correct.
o Investing in large capital projects by the government, with the aim of creating more jobs and
providing more people with spending power. Hence option 4 is correct.
• "Reflation trade” has been a common buzzword in the investment community after the passage of
blockbuster policies during the onset of the 2020 coronavirus pandemic, which resulted in significant
injections of liquidity into the financial markets. Reflation trade refers to securities/sectors that benefit
from reflation. The securities/sectors benefit from faster economic growth and pricing pressure, such as
the cyclical sectors (banks, energy producers, etc.).

Q 70.C
• Recent context: Reserve Bank of India (RBI) has issued a framework for acceptance of ‘Green
Deposits’.
• Green deposit:
o An interest-bearing deposit, received by regulated entities (REs) for fixed period and proceeds of
which are earmarked for being allocated towards green finance.
o Aims to offer green deposits to customers, protect interest of depositors, to achieve their sustainability
agenda, address greenwashing concerns and augment the flow of credit to green activities/project.
o Applicable to following entities (known as regulated entities (RE)):
✓ Scheduled Commercial Banks including Small Finance Banks (excluding Regional Rural Banks,
Local Area Banks and Payments Banks)
✓ All Deposit taking Non-Banking Financial Companies (NBFCs) registered with RBI including
Housing Finance Companies (HFCs). Hence statement 1 is correct.
o To be denominated in Indian Rupees only.
o Not mandatory for RE to raise green deposits.
o RE cannot finance green activities/ projects first and raise green deposits later.
o Investments made by REs in Sovereign Green Bonds are covered under the framework. Hence
statement 2 is correct.
o Deposits raised under the framework are covered by the Deposit Insurance and Credit Guarantee
Corporation.
o Green activities/ projects financed under the framework can be classified under the priority sector if
they meet priority sector lending (PSL) guidelines of RBI. Hence statement 3 is correct.
o Banks are allowed to offer overdraft facilities to customers against Green Deposits.
o Annual Independent Third-Party Verification/Assurance for funds raised through green deposits.

30 www.visionias.in ©Vision IAS

Telegram: UPSC_SARTHI
Q 71.D
• The Internal Benchmark Lending Rates are a set of reference lending rates which are calculated after
considering factors like the bank's current financial overview, deposits and non performing assets (NPAs)
etc. BPLR, Base rate, MCLR are the examples of Internal Benchmark Lending Rate.
• To ensure complete transparency and standardization, RBI mandated the banks to adopt a uniform
external benchmark within a loan category, effective 1st October, 2019. As name suggests, external
benchmark means the lending rate not internally set by individual banks based on internal benchmarks but
rather external benchmarks.
• Unlike MCLR which was internal system for each bank, RBI has offered banks the options to choose
from 4 external benchmarking mechanisms:
o The RBI repo rate
o The 91-day T-bill yield
o The 182-day T-bill yield
o Any other benchmark market interest rate as developed by the Financial Benchmarks India Pvt. Ltd.
• Banks are free to decide the spread over the external benchmark. However, credit risk premium may
undergo change only when borrower’s credit assessment undergoes a substantial change, as agreed upon
in the loan contract. Further, other components of spread including operating cost could be altered once in
three years.
• The interest rate under external benchmark shall be reset at least once in three months.
• Hence option (d) is the correct answer.

Q 72.C
• Gross capital formation is measured by the total value of the gross fixed capital formation, changes in
inventories and acquisitions less disposals of valuables. Gross fixed capital formation is measured by the
total value of a producer's acquisitions, less disposals, of fixed assets during the accounting period plus
certain additions to the value of non-produced assets realised by the productive activity of institutional
units. Fixed assets are tangible or intangible assets produced as outputs from processes of production that
are themselves used repeatedly or continuously in other processes of production for more than one year.
• Tangible assets are physical items owned by a company, such as equipment, buildings, and inventory.
Tangible assets are the main type of asset that companies use to produce their products and services.
Intangible assets are nonphysical items that have a monetary value because they represent potential
revenue. Intangible assets include patents, copyrights, and a company's brand.
• There is substantial diversity in the different types of gross fixed capital formation that may take place.
The following main types may be distinguished:
o Acquisitions, less disposals, of new or existing tangible fixed assets, subdivided by type of asset into:
(i) Dwellings;(ii) Other buildings and structures; (iii) Machinery and equipment; (iv) Cultivated assets
- trees and livestock that are used repeatedly or continuously to produce products such as fruit, rubber,
milk, etc.; Hence option 1 is correct.
o Acquisitions, less disposals, of new and existing intangible fixed assets, sub-divided by type of asset
into: (i)Mineral exploration; (ii) Computer software; (iii) Entertainment, literary or artistic originals;
(iv) Other intangible fixed assets; Hence option 2 is correct.
o Major improvements to tangible non- produced assets, including land; Hence option 3 is correct.
o Costs associated with the transfers of ownership of non-produced assets.
Q 73.B
• The Agreement on Trade-Related Investment Measures (TRIMS) recognizes that certain investment
measures can restrict and distort trade. It states that WTO members may not apply any measure that
discriminates against foreign products or that leads to quantitative restrictions, both of which violate basic
WTO principles. All WTO member countries are parties to this Agreement. Hence statement 3 is
correct.
• The TRIMs Agreement prohibits certain measures that violate the national treatment and
quantitative restrictions requirements of the General Agreement on Tariffs and Trade (GATT). A
list of prohibited TRIMS, such as local content requirements, is part of the Agreement. Hence statement
2 is correct.
• The TRIMS Committee monitors the operation and implementation of the Agreement and allows
members the opportunity to consult on any relevant matters.
• This Agreement, negotiated during the Uruguay Round, applies only to measures that affect trade
in goods. The TRIMs Agreement does not cover services. Hence statement 1 is not correct.
31 www.visionias.in ©Vision IAS

Telegram: UPSC_SARTHI
Q 74.C
• In a statement issued after International Energy Agency (IEA) 2024 ministerial meeting in Paris, the
agency said the talks with India are in recognition of the country’s “strategic importance” in tackling
global energy and climate challenges. India, which joined the IEA as an associate member in 2017, sent a
formal request for full membership in October 2023.
• IEA is an international intergovernmental organization based in Paris that was established in 1974. Its
stated mandate is to maintain the stability of the international oil supply. The IEA operates within the
broader framework of the Organization for Economic Co-Operation and Development (OECD). The IEA
is made up of 31 member countries. It also includes eight associate countries.
• Criteria for membership: A candidate country for the IEA must be a member country of the
OECD. Hence option 4 is not correct.
• In addition, it must demonstrate several requirements. These are:
o Crude oil and/or product reserves are equivalent to 90 days of the previous year’s net imports,
to which the government has immediate access (even if it does not own them directly) and could be
used to address disruptions to global oil supply. Hence option 1 is correct.
o A demand restraint programme to reduce national oil consumption by up to 10%. Hence option 2 is
correct.
o Legislation and organisation to operate the Coordinated Emergency Response Measures (CERM) on a
national basis.
o Legislation and measures to ensure that all oil companies under its jurisdiction report information
upon request. Hence option 3 is correct.
o Measures are in place to ensure the capability of contributing its share of an IEA collective action.

Q 75.A
• The functions of the Deposit Insurance and Credit Guarantee Corporation (DICGC) are governed by the
provisions of 'The Deposit Insurance and Credit Guarantee Corporation Act, 1961' (DICGC Act) and
'The Deposit Insurance and Credit Guarantee Corporation General Regulations, 1961' framed by the
Reserve Bank of India.
• All commercial banks including branches of foreign banks functioning in India, local area banks and
regional rural banks are insured by the DICGC. At present all co-operative banks are covered by the
DICGC. Primary cooperative societies are not insured by the DICGC.
• The DICGC insures all deposits such as savings, fixed, current, recurring, etc. deposits except the
following types of deposits. Hence option 1 is correct.
o Deposits of foreign Governments; Hence option 3 is not correct.
o Deposits of Central/State Governments; Hence option 4 is not correct.
o Inter-bank deposits; Hence option 2 is not correct.
o Deposits of the State Land Development Banks with the State co-operative bank;
o Any amount due on account of and deposit received outside India• Any amount, which has been
specifically exempted by the corporation with the previous approval of Reserve Bank of India
• Each depositor in a bank is insured upto a maximum of ₹ 5,00,000 (Rupees Five Lakhs) for both
principal and interest amount held by him in the same right and same capacity as on the date of
liquidation/cancellation of bank's licence or the date on which the scheme of amalgamation/merger/
reconstruction comes into force.
Q 76.B
• The Reshmi Rumal Tehreek, also known as the Silk Letter Movement (1913-1920), emerged as a
significant initiative led by Deobandi leaders to challenge the dominance of the British Empire.
Hence option (b) is the correct answer.
o Metaphorically, it aimed to weaken British rule by enveloping it in the soft embrace of silk fabric. A
prominent institution that emerged during this time was Darul Uloom Deoband, founded in 1866.
• This educational establishment played a pivotal role in producing influential leaders like Maulana
Mahmood Hassan (also known as Sheikh ul Hind) and Mohammed Mian Mansoor Ansari, who became
pioneers of the movement.
• The Silk Letter Movement referred to as Tehreek-e-Reshmi Rumal derived its name from the unique
communication method used by Shaikh ul Hind and other Deobandi leaders.
o They exchanged confidential information through written correspondence on pieces of silk fabric.
These letters contained strategic plans, details about weapons and ammunition, recruitment strategies,
and guidance for training volunteers.
32 www.visionias.in ©Vision IAS

Telegram: UPSC_SARTHI
• The movement attracted numerous enthusiastic young individuals and gained significant
momentum. It evolved beyond a gathering of like-minded activists into a fully-fledged armed
movement with international support and sponsors.
• Nations opposing British rules, such as Turkey, Afghanistan, Russia, and Imperial Germany,
provided essential funds, arms, and ammunition for the cause. However, the covert nature of the
plot was exposed when the Punjab CID intercepted a collection of letters.
• These correspondences, written by Ubaidullah Sindhi, a Deobandi leader in Afghanistan, were intended
for Mahmud Hasan Deobandi, another significant figure residing in the Hejaz. These vital
communications were penned on silk cloth, giving rise to the movement's unique name.
• Unfortunately, with the arrest of key plotters, their efforts were in vain. The circumstances leading to this
failure remained unexplained. Using the intercepted letters as evidence, the British swiftly apprehended
numerous influential leaders. A total of 222 individuals underwent intense interrogations, intimidation,
and torture while in custody.

Q 77.C
• The preparations for the first Congress had begun very early in 1885. It was decided to hold a
conference at Poona from 25 to 31 December. The Poona Sarvajanik Sabha had completed all
arrangements for the intended Conference. The outbreak of the cholera epidemic on the eve of the
conference led to the shift of its first session from Poona to Bombay.
• Allan Octavian Hume (1829-1912) was the son of Joseph Hume, the latter was a radical member of the
House of Commons. Allan O. Hume was a member of the Indian Civil Service and served in top
administrative positions in India from 1849 to 1882. He wished to relieve the Indian masses from poverty,
misery, and exploitation.
• He was the prime mover in the foundation of the Indian National Congress in 1885 and worked as its
General Secretary from 1885 to 1906. Besides the foundation of the Congress, Hume has demonstrated
great interest in ornithology, horticulture, theosophy, and reforms. Never was there a British, who
nurtured the cause of Indian regeneration and nationalism with utmost devotion and care, except A.O.
Hume. Hence option (c) is the correct answer.

Q 78.D
• The Bengal Volunteers emerged as an underground revolutionary group opposing British rule in India,
operating from its inception in 1928 until Indian independence.
• This group was formed during the 1928 Calcutta session of the Indian National Congress under the
leadership of Subhas Chandra Bose. Major Satya Gupta was appointed as the head of the group,
with Subhas Chandra Bose serving as the General Officer Commanding (GOC). Hence statement 1
is not correct.
• The Bengal Volunteers continued their activities beyond the Kolkata Congress session and soon
transformed into an active revolutionary association.
• In the early 1930s, members of the Bengal Volunteers decided to launch 'Operation Freedom.'
Initially, this operation was initiated to protest against police brutality in various Bengal jails.
o In August 1930, the revolutionary Bengal Volunteers devised a plan to assassinate the then
Inspector General of Police, Lowman, who was visiting the Medical School Hospital. Hence
statement 2 is not correct.
o On August 29, 1930, Benoy Basu, dressed in traditional Bengali attire, bypassed security and fired at
Lowman at close range, killing him instantly, while Superintendent of Police Hodson was seriously
injured.
o The next target of the Bengal Volunteers was the Inspector General of Prisons, Col N.S. Simpson is
notorious for his harsh treatment of prisoners.
o The revolutionaries not only planned to assassinate him but also intended to strike fear within British
official circles by launching an attack on the Secretariat Building, known as the Writers' Building, in
Dalhousie Square, Kolkata.
o On December 8, 1930, Benoy Basu, along with Dinesh Gupta and Badal Gupta, dressed in European
attire, entered the Writers' Building and killed Simpson.
• The Bengal Volunteers, under the leadership of Netaji Subhas Chandra Bose, made a significant
contribution to the Indian freedom movement. Their actions created a substantial impact on the British
administration, ing a shift in policies regarding administrative operations. This continued until the year
1947.
33 www.visionias.in ©Vision IAS

Telegram: UPSC_SARTHI
Q 79.C
• The Satavahana Empire was an ancient Indian dynasty that ruled in the Deccan region. The decline
of the Satavahana Empire in the Deccan region contributed to the emergence of several
successor states and regional powers. As the Satavahana influence waned, various smaller dynasties and
kingdoms rose to prominence in different parts of the Indian subcontinent. Some notable successor states
include:
• In coastal Andhra, there was a succession of families like the Ikshvakus, the Salankayanas, and
others. In Karnataka, the most important ruling family was that of the Kadambas. The Kadamba power
was founded by Brahmin Mayurasarman whose Talagunda inscription gives some interesting details
of the circumstances leading to the establishment of the kingdom and also some idea regarding its
extent. Hence options 1, 2, and 3 are correct.
• Several small kingdoms arose in the different parts of Maharashtra. The Puranas mention the Abhiras,
Gardabhins, Yavanas, Tusaras, Sakas, Murundas, Maunas, and Kilakilas as the successors of the
Andhras i.e. the Satavahanas. Hence option 4 is correct.
• The Karakota Dynasty ( 625 − 855 CE) ruled over the Kashmir valley and some northern parts of the
Indian subcontinent during the 7th and 8th centuries. Hence option 5 is not correct.
• Ikshvakus: The Ikshvaku kingdom flourished in Andhra Pradesh, near the eastern Krishna Valley for
about one hundred years, from 220 to 320 CE. It patronized Buddhism, Brahminism, as well as older folk
religions. Their capital city, Vijayapuri, had at least 18 Brahminical temples, mostly along the Krishna
riverfront.
• The Salankayanas: This dynasty of ancient India ruled a part of the Andhra Pradesh region in India from
300 to 440 CE. Their territory was located between the Godavari and the Krishna rivers. Their capital was
located at Vengi, modern Pedavegi near Eluru in the West Godavari district of Andhra Pradesh.
• The Kadambas: The Kadambas were an ancient royal family of Karnataka, India, that ruled northern
Karnataka and the Konkan from Banavasi in present-day Uttara Kannada district. The kingdom was
founded by Mayurasharma in c. 345, and at later times showed the potential of developing into imperial
proportions.
• The Abhiras: The Abhira dynasty was a dynasty that ruled over the western Deccan, where it perhaps
succeeded the Satavahana dynasty. From 203 CE to roughly 270 or 370, this dynasty formed a vast
kingdom. The Abhira era was started by Ishwarsena in AD 249.

Q 80.A
• The Madai Mela in Bastar, Chhattisgarh begins when during agand mahina, ‘dhaan katai’ is
complete (rice harvest). The harvesting season ends and soon after the madai mela begins. Hence
pair 1 is not correctly matched.
o After a good harvest, it is a form of thanking the gods and goddesses. It is also held to dispel the
yearlong struggles, efforts, and tiredness through music, dance, and interacting with the community.
o The communities that celebrate the Madai Festival are Kurna and Charama of the Kanker district,
most of the tribes of the Bastar region, people of Narayanpur, Kondagaon, Bhanupratappur, Antagarh
and Pakhanjore.
• The Karaga festival is held at the Dharmaraya Swamy Temple in Bangalore, Karnataka. Just after
dusk on the Karaga day between March and April, a priest dressed in female attire leads a
spectacular procession, to the accompaniment of dazzling swordplay by a number of dhoti-clad,
bare-chested Thigalars. Hence pair 2 is correctly matched.
o On his head, he carries a flower-bedecked pyramid. A unique feature of the Karaga is the unbroken
tradition of visiting the tomb of an 18th-century Muslim saint every year – this custom has become a
symbol of Hindu-Muslim unity.
o Celebrated annually, the 300-year-old Karaga festival has its roots in the Hindu mythological epic
Mahabharata
o As part of the celebrations, devotees take out a large procession, carrying earthen pots on their heads.
o Karaga festival is celebrated each year to mark the return of Draupadi in the form of Adishakti.
• Moatsu festival is one of the most eagerly awaited festivals in the beautiful state of Nagaland.
Making the entire state lively with its vibrant festivities, Moatsu Mong festival is celebrated by Ao
tribe of the state. Hence pair 3 is not correctly matched.
o Moatsu festival is celebrated in the hope of a good harvest. The tribals observe this festival after
sowing seeds in the fields and believe that it brings people together and encourage bonding between
them. This three-day festival is celebrated with great fanfare.
34 www.visionias.in ©Vision IAS

Telegram: UPSC_SARTHI
o It witnesses many customs and rituals performed before sowing is done in the fields and is a much-
deserved break for them after going through various farm-related activities like burning jungles,
clearing fields, removing weeds and wild plants, cleaning up wells or Tsubu, doing repair work in
their homes, and so on.
o The natural customary practice of the forefathers was competing in making the best rice-beer and
rearing the best possible pigs and cows to be slaughtered during the festival.
• Hence option (a) is the correct answer.

Q 81.B
• During the Gupta period revenue generated from agriculture was the main source of income for the state.
For the collection of this revenue, a large section of officials was appointed.
• Contemporary sources (such as Amarakosha) give information about the diverse types of crops produced
in this period such as paddy, cotton, oilseeds, indigo, mustard seeds, etc.
• The people of this period had good knowledge of agriculture and they were aware of the different types of
soils. In this regard Amarakosha gives us information about twelve types of soils — urvara
(fertile), ushara (dry), maru (desiccated), aprahata (untilled), sadbala (grassy land), pankila (covered
with mud), jalaprayamanupam (land located near a source of water), kachcha (marshy), sarkara, sarkati,
nadimatrika (well-watered by rivers) and devematrika (area well fed by rain).
• The cultivated land was called as kshetra and uncultivated land was called by various names such as khila
or aprahata.
• Hence option (b) is the correct answer.

Q 82.B
• Slavery was permitted in Islam and it was prevalent in the Islamic world.
• The success of a campaign was to be measured by the number of captives acquired along with gold, silver,
cattle, and horses.
• Qutbuddin Aibak captured 20 thousand slaves in his Gujarat campaign of 1195 and 50 thousand slaves in
the raid on Kalinjar in 1202.
• Even after the establishment of the Sultanate, the enslavement continued through campaigns in yet-
to-be-conquer areas. One of the main objects of Balban"s raid of Ranthambor and Malik Kafur's
campaigns in the Deccan was to get slaves. Hence statement 1 is not correct.
• Another source of getting captives was the plunder raids of rebellious villages (mawas) in the Sultanate
that refused to pay the kharaj or tribute. The number of slaves received from these sources was enormous.
• There were 50,000 slaves in Alauddin Khilji's (1296,1316) establishment: The number increased to
1,80,000 under Feroz Tughluq (135.1-88). Besides the Sultans, nobles had their private large retinues of
slaves including a large number of concubines. Even the respectable poor kept slaves.
• There was a large slave market. The prices of slaves of the two sexes were fixed under Alauddin
Khalji as recorded by Barani. The abundance of slaves encouraged the continuous export of slaves
from India to the Islamic World. Hence statement 2 is correct.
Q 83.B
• Krishnadevaraya (17 January 1471 — 17 October 1529) was an emperor of the Vijayanagara Empire
reigning from 1509 to 1529. He was the third monarch of the Tuluva dynasty and is considered to be one
of the greatest rulers in Indian history. He ruled the largest empire in India after the fall of the Islamic
Delhi Sultanate. Presiding over the empire at its zenith, he is regarded as an icon by many Indians.
Krishnadevaraya earned the titles Andhra Bhoja (lit. "Bhoja of Andhra"), Karnatakaratna
Simhasanadeeshwara (lit. "Lord of the Jewelled Throne of Karnataka"), Kannada Rajya Rama Ramana
(lit. "Lord of the Kannada Empire). He became the dominant ruler of the peninsula by defeating the
sultans of Bijapur, Golconda, the Bahmani Sultanate, and the Gajapatis of Odisha, and was one of the
most powerful Hindu rulers in India.
• Pratap Singh I (circa 9 May 1540 – 19 January 1597) popularly known as Maharana Pratap, was a king
of Mewar, a region in north-western India in the present-day state of Rajasthan. He is notable for leading
the Rajput resistance against the expansionist policy of the Mughal Emperor Akbar including the Battle of
Haldighati and Battle of Dewair which have turned him into a hero.
• Babur (14 February 1483 – 26 December 1530; born Zahīr ud-Dīn Muhammad) was the founder of
the Mughal Empire in the Indian subcontinent. He was a descendant of Timur and Genghis Khan through
his father and mother respectively. He was also given the posthumous name of Firdaws Makani
('Dwelling in Paradise'). Hence option (b) is the correct answer.
35 www.visionias.in ©Vision IAS

Telegram: UPSC_SARTHI
• Jalal-ud-din Muhammad Akbar, popularly known as Akbar (1542 - 1605), and also as Akbar I was the
third Mughal emperor, who reigned from 1556 to 1605. Akbar succeeded his father, Humayun, under a
regent, Bairam Khan, who helped the young emperor expand and consolidate Mughal domains in the
Indian subcontinent.
• Jahangir (1569 - 1627): Nur-ud-Din Muhammad Salim (31 August 1569 – 28 October 1627) known
by his imperial name Jahangir ( 'Conqueror of the World'), was the fourth Mughal Emperor, who ruled
from 1605 until his death in 1627.

Q 84.C
• Bhand Pather, the traditional theatre form of Kashmir, is a unique combination of dance, music
and acting. Satire, wit and parody are preferred for inducing laughter. In this theatre form, music
is provided with surnai, nagaara and dhol. Since the actors of Bhand Pather are mainly from the
farming community, the impact of their way of living, ideals and sensitivity is discernible. Hence
option (c) is the correct answer.
• Bhaona is a presentation of the Ankia Naat of Assam. In Bhaona cultural glimpses of Assam, Bengal
Orissa, Mathura and Brindavan can be seen. The Sutradhaar, or narrator begins the story, first in Sanskrit
and then in either Brajboli or Assamese.
• Dashavatar is the most developed theatre form of the Konkan and Goa regions. The performers
personify the ten incarnations of Lord Vishnu god of preservation and creativity. The ten incarnations are
Matsya (fish), Kurma (tortoise), Varaha (boar), Narsimha (lion-man), Vaman (dwarf), Parashuram, Rama,
Krishna (or Balram), Buddha and Kalki. Apart from stylized make-up, the Dashavatar performers wear
masks of wood and papier mache.
• Maach is the traditional theatre form of Madhya Pradesh. The term Maach is used for the stage itself
as also for the play. In this theatre form songs are given prominence in between the dialogues. The term
for dialogue in this form is bol and rhyme in narration is termed vanag. The tunes of this theatre form are
known as rangat.

Q 85.A
• Humayun acquired a love of the art of painting at the court of Shab Tahmasp of Persia. Humayun was so
influenced by the art practiced there that he commissioned Mir Syed Ali and Khwaja Abdus two Persian
masters, to illustrate manuscripts for him. These two painters joined Humayun's entourage on his
triumphant return to India. Hence statement 2 is not correct.
o An important painting from Humayun's period is titled 'Princes of the House of Timur'.
• The emergence of the Mughal School of painting as distinct from all other styles was mainly due to the
deep interest Akbar took in the promotion of this art.
• The first major project undertaken during Akbar's regime was that of illustrating the Hamza
Nama. It began m 1562 for which several artists were employed at the court. Hence statement 3 is
not correct.
• The Mughal style became recognizable within a span of fifteen years since the setting up of royal atelier
under Akbar. In the next decade or so, i.e. by about 1590s it acquired a distinctive form which was
marked by.
o naturalism & rhythm
o clothing objects of daily use assuming Indian forms.
o picture space having subsidiary scenes set in the background
o extraordinary vigor of action and violent movement
o luxuriant depiction of foliage & brilliant blossoms
• Jahangir took a deep interest in painting even as a prince. He maintained his own studio apart from
Akbar's large atelier. Jahangir's preference was for paintings of hunting scenes, birds and flowers.
o In the period of Jahangir's rule manuscripts became less important than individual pictures.
o Jahangir's paintings seen to accentuate a formalist style, i.e., making the work realistic and preferring
the precise recording of contemporary reality.
o The paintings of this period have broad margins which are gorgeously decorated with the
depiction of flora and faces of human figures, etc. designs from plant motifs. Hence statement 1
is correct.
• Under Shah Jahan, the colours of the paintings were more decorative and gold was more frequently used
for embellishments.

36 www.visionias.in ©Vision IAS

Telegram: UPSC_SARTHI
• Mughal painting was also influenced by the European art. Some of the themes of European art were
incorporated by Mughal painters and they also adopted a few of the techniques of European artists.
o One important feature that becomes noticeable due to European influence in some Mughal paintings is
the attempt to make them three dimensional.
o The depiction of motits like winged angles and roaring clouds in Mughal paintings was again
under the influence of European paintings.

Q 86.D
• Technically, LDPE can be recycled. However Plastic bags, for example, tend to tangle in recycling
machinery risking endangering the entire recycling process. As compared to LDPE, it is more easy to
recycle HDPE. Hence statement 1 is not correct.
• Polystyrene (PS) is mainly used in food packaging and insulation. Polystyrene foam is 95% air so it is
not cost-effective to store or ship. It is often contaminated with food or drink, and it is difficult to clean
because it is so porous. Hence it is not feasible to recycle polystyrene on a commercial scale.
• Polyethylene terephthalate (better known as PET). It is probably the easiest to recyclemost. It is used in
the commercialization of countless products in sectors such as food, automotive, textile or
pharmaceutical. Hence statement 2 is not correct.
• The unique challenges associated with PVC are, one, that it releases hydrochloric acid upon heat
treatment which is corrosive to the recycling equipment. Further PVC are usually formulated with
a lot of plasticizer additives like Cell A plasticizers which are very toxic. Once it gets into the
recycling stream it contaminates other plastics.Hence statement 3 is not correct.
• Hence option (d) is the correct answer.

Q 87.C
• Deer is one of the most beautiful creatures on planet earth and extends to about 34 species. A lot of them
inhabit India:
o Spotted Deer or Chital deer is the most common native deer species in the Indian subcontinent.
Spotted Deers are found in numbers in India, Nepal, Bangladesh, Sri Lanka, and Pakistan. It is also
known as Axis Deer or Asiatic Deer. According to the IUCN Red List assessment, Spotted Deer are
categorized as Least concerned. Spotted Deers are sexually dimorphic. Males are significantly larger
than females, and only male deers possess antlers. Hence option 1 is correct.
o Kasturi Mrig or Musk Deer: Out of 7 species of musk deer found in Asia only 5 are found in India.
They are found mostly in Himalayan Region. Carried only by the male deer in its abdomen, the musk
emits a sweet persistent aroma and is highly valued for its cosmetic and alleged pharmaceutical
properties. Hence option 2 is correct.
o Barasingha also known as the Swamp deer: Only the male species of Barasingha have antlers. It
is the branched and interconnected antlers that give the Barasingha its unique look. Only the
male species of Barasingha have antlers. The Barasingha or Swamp Deer is one of the deer species
that is found across India’s national parks and wildlife sanctuaries. It is most popular at the Kanha
National Park in Madhya Pradesh and the Kaziranga National Park in Assam where it is found in huge
numbers. Hence option 3 is correct.
• Barking Deer: Barking deer is also known as muntjacs and is found in the lower Himalayas. Their call
sounds like a barking dog, and hence the name. Hence option 6 is correct.
• Himalayan Musk Deer: It is a small dog-like ungulate with no antlers.
• Indian Hog Deer: It is widely distributed theough the plains of India particularly in Keibul Lamjao NP,
Kaziranga NP and Manas National Park.
• Mouse Deer: India's smallest deer, it is found naturally in Kanha NP, Nagarhole NP and parts of Western
Ghats. Hence option 5 is correct.
• The reindeer or caribou is a species of deer with circumpolar distribution, native to Arctic, subarctic,
tundra, boreal, and mountainous regions of Northern Europe, Siberia, and North America. Hence option 4
is not correct.
• Hence option (c) is the correct answer.

Q 88.C
• Crustaceans (make up a very large group of the Arthropods which include the crabs, lobsters, crayfish,
shrimp, krill, barnacles brine shrimp, copepods, ostracods and mantis shrimp. Crustaceans are found in a
wide range of habitats - most are free-living freshwater or marine animals, but some are terrestrial (e.g.
37 www.visionias.in ©Vision IAS

Telegram: UPSC_SARTHI
woodlice), some are parasitic (e.g. fish lice) and some do not move (e.g. barnacles). Crustaceans are a
group of arthropods that are a part of the subphylum Crustacea.
o Crustaceans are invertebrates with a hard exoskeleton (carapace), a segmented body that is bilaterally
symmetrical, more than four pairs of jointed appendages ("legs") and an open circulatory system (the
"blood" does not flow in a closed loop).
o They also have eyes usually on stalks, a primitive ventral nerve cord and "brain" (ganglia near the
antennae), a digestive system which is a straight tube for grinding food and a pair of digestive glands.
o Gills are used for respiration and they have a pair of green glands to excrete wastes (found near the
base of the antennae).
o Their bodies are composed of three body segments - the head, the thorax and the abdomen.
o Most Crustaceans are either male or female and reproduce sexually. A small number, including
barnacles, are hermaphrodites. In other species, viable eggs are produced by a female without needing
to be fertilised by a male.
• Mollusca is one of the most diverse groups of animals on the planet, with at least 50,000 living species
(and more likely around 200,000). It includes such familiar organisms as snails, octopuses, squid, clams,
scallops, oysters, and chitons.
o Molluscs are a clade of organisms that all have soft bodies which typically have a "head" and a "foot"
region. Often their bodies are covered by a hard exoskeleton, as in the shells of snails and clams or the
plates of chitons.
o They range in distribution from terrestrial mountain tops to the hot vents and cold seeps of the deep
sea, and range in size from 20-meter-long giant squid to microscopic aplacophorans, a millimeter or
less in length, that live between sand grains.
• Crab is a crustacean and all others are Mollusks. Hence option (c) is the correct answer.

Q 89.D
• Keystone Species: Every ecosystem has certain species that are critical to the survival of the other species
in the system. The keystone species could be a huge predator or an unassuming plant, but without them
the ecosystem may not survive. There are three types of keystone species cited by many scientists:
predators, ecosystem engineers, and mutualists.
o Predators: They help control the populations of prey species, which in turn affects the quantity of
plants and animals further along the food web. For example: shark, tiger, lion etc.
o An ecosystem engineer is an organism that creates, changes, or destroys a habitat. There is perhaps
no clearer example of a keystone engineer than the beaver.
o Mutualists: When two or more species in an ecosystem interact for each other’s benefit, they are
called mutualists. Bees are a primary example of this. As bees take the nectar from flowers, they
collect pollen and spread it from one flower to the next, enhancing the odds of fertilization and greater
flower growth. Nectar and pollen are also the primary food sources for the bees themselves. Hence
pair 1 is correctly matched.
• A rare alternative term for a species that is endemic is "precinctive", which applies to species (and
other taxonomic levels) that are restricted to a defined geographical area.
o Lion-tailed Macaque is endemic to the Western Ghats in the states of Karnataka, Kerala and Tamil
Nadu. Although the species has a relatively wide range, its area of occupancy is small and severely
fragmented. Hence pair 2 is correctly matched.
✓ Primarily diurnal arboreal, it prefers the upper canopy of primary tropical evergreen rainforest.
✓ It can also be found in monsoon forests in hilly country and in disturbed forest.
✓ IUCN Red list: Endangered.
✓ It is listed in Appendix I of CITES.
✓ It is protected under Schedule I, Part I, of the Wildlife (Protection) Act, 1972.
• Indicator species are those flora and fauna which define a particular trait or change in the environment.
Such species are sensitive to environmental change and can act as early warning regarding adverse
environmental changes in particular area. For example, Frogs, Lichens, Salmon etc. Hence pair 3 is
correctly matched.
• Invasive alien species are plants, animals, pathogens and other organisms that are non-native to an
ecosystem, and which may cause economic or environmental harm or adversely affect human health. For
example, West Indian Lantana, Eucalyptus, Wattle, Common Water Hycinth,
o Prosopis juliflora is an invasive alien plant species, is known as a threat to biodiversity and ecosystem
services world over. Hence pair 4 is correctly matched.

38 www.visionias.in ©Vision IAS

Telegram: UPSC_SARTHI
Q 90.D
• The carbon cycle involves the exchange of carbon between the atmosphere and organisms. It is the
biogeochemical cycle by which carbon is interchanged among the biosphere, hydrosphere, atmosphere,
and geosphere of the Earth. It is a gaseous cycle.
• As our planet and its atmosphere form a closed environment, the amount of carbon in this system
does not change. Wherever the carbon is located in the atmosphere or on Earth, it is constantly in flux or
motion. Hence statement 3 is not correct.
• Most carbon on earth is stored in rocks and sediments, while the rest is located in the ocean,
atmosphere, and living organisms. These are the reservoirs, or sinks, through which the carbon cycles.
Hence statement 2 is not correct.
o Carbon from the atmosphere is taken up by the green plants by photosynthesis and then to animals by
the food chain.
o By respiration and decomposition of dead organic matter, it returns to the atmosphere.
• Although widely distributed in nature, carbon is not particularly plentiful—it makes up only about
0.025 percent of Earth's crust—yet it forms more compounds than all the other elements combined. The
most abundant element in the earth's crust is oxygen (46. 1%). Silicon is second, making up 28%,
followed by aluminum (8%), iron (5%), magnesium (2%), calcium (4%), sodium (3%), and potassium
(3%). Hence statement 1 is not correct.

Q 91.C
• Global dimming is a phenomenon which produces forces that act opposite to global warming in nature.
Global dimming reduces the amount of sun’s rays reaching the earth’s atmosphere causing a drop of
temperatures around the globe. Hence statement I is correct.
• Global dimming is not thought to be due to changes in the sun's luminosity, as these have been too small
to explain the magnitude of dimming observed. Instead, air pollution from human activity is thought to be
the major contributor. Aerosols which form from pollution can directly reflect and absorb radiation before
it reaches the planet's surface and make clouds brighter and longer lasting, meaning they reflect more
sunlight. Hence statement II is not correct.
• What causes of Global Dimming?
o Aerosols (colloid of fine solid particles or liquid droplets, in air or other gas) have been found to be
the major cause of global dimming. Most aerosols in the atmosphere only scatter light from the sun,
sending some of the sun's radiant energy back to space and exerting a cooling influence on Earth's
climate.
o Particle matters like sulphur dioxide, soot and ash (by-product of burning of fossil fuels by industry
and internal combustion engines) enter the atmosphere and directly absorbs solar energy and reflects
radiation back into the space, before it reaches the planet’s surface.
o Water droplets containing airborne particles like sulphur dioxide, soot and ash (by-product of burning
of fossil fuels by industry and internal combustion engines) form polluted clouds. These polluted
clouds have heavier and larger number of droplets. These changed properties of the cloud – such
clouds are called ‘brown clouds’ – makes them more reflective.
o Vapours emitted from the planes flying high in the sky called contrails are another cause of heat
reflection and related global dimming.
• What are the effects of Global dimming?
o Just like Global Warming, Global dimming has also withering effects on the atmospheric temperature
of the earth as well as on the living beings.
✓ It reduces the atmospheric temperature of the Earth. As a result, precipitation will be low that
causes very low rainfall, which leads to droughts.
✓ Pollutants and by-products cause thick fog, acid rain and pollution, causing many diseases such as
respiratory diseases.
✓ Cooling effects causes decline of vegetation, soil erosion etc.
• Global dimming Vs Global warming:
o Global dimming is the reduction in heat or the reduction in the sun’s rays reaching the earth whereas
Global Warming is the increase in Earth's mean surface temperature.
o Global dimming covers the drastic effects of global warming.

39 www.visionias.in ©Vision IAS

Telegram: UPSC_SARTHI
Q 92.A
• India’s leopard numbers rose by 8% from 12,852 in 2018 to 13,874 in 2022, according to a report made
public by the Environment Ministry recently.
• While the highest number of leopards were reported in Madhya Pradesh (3,907), only three other States
reported over a 1,000 animals each Maharashtra (1,985), Karnataka (1,879) and Tamil Nadu (1,070).
Hence statement 1 is not correct.
• Out of total 13874 leopards in India 8841 leopards are concentrated in 4 states namely Madhya Pradesh,
Maharashtra, Karnataka and Tamil Nadu. Hence statement 2 is correct.
• Forest surveyors cover nearly 6.5 lakh km looking for carnivore signs across 20 States; human-wildlife
conflict is the likely reason for decline in the population by 22% in Uttarakhand. While Uttarakhand
reported a 22% decline in the numbers — reportedly due to poaching and man-animal conflict, Arunachal
Pradesh, Assam and West Bengal saw a collective 150% rise to 349 animals. Hence statement 3 is not
correct.
• Hence option (a) is the correct answer.

Q 93.A
• Prokaryotic cells and eukaryotic cells are two primary types of cells that make up living organisms.
The key differences between them are:
• Nucleus:
o Prokaryotic Cells: Lack of a true nucleus. The genetic material (DNA) is present in a nucleoid
region, not enclosed by a membrane.
o Eukaryotic Cells: Have a true nucleus enclosed by a nuclear membrane, separating the genetic
material from the cytoplasm. Hence, statement 1 is correct.
• Membrane-bound Organelles:
o Prokaryotic Cells: Lack membrane-bound organelles. Some structures like ribosomes are present,
but they are not surrounded by membranes.
40 www.visionias.in ©Vision IAS

Telegram: UPSC_SARTHI
o Eukaryotic Cells: Contain membrane-bound organelles such as the endoplasmic reticulum, Golgi
apparatus, mitochondria, chloroplasts (in plant cells), lysosomes, and others.
• Cell Size:
o Prokaryotic Cells: Generally smaller in size, typically ranging from 0.5 to 5 micrometers.
o Eukaryotic Cells: Larger and more diverse in size, ranging from 10 to 100 micrometers. Hence,
statement 2 is not correct.
• DNA Structure:
o Prokaryotic Cells: DNA is usually a single, circular molecule located in the nucleoid. Hence
statememt 3 is not correct.
o Eukaryotic Cells: DNA is organized into multiple linear chromosomes found in the nucleus.
• Cell Division:
o Prokaryotic Cells: Reproduce through binary fission, a simpler form of cell division.
o Eukaryotic Cells: Undergo mitosis during cell division, a more complex process involving multiple
stages.
• Hence, option (a) is the correct answer.

Q 94.B
• Weight is the product of mass(m) and acceleration due to gravity(g) i.e. mg. While moving from the
Equator towards the poles the value of acceleration due to gravity (g) increases as it is inversely
proportional to the radius of the earth and the radius of the earth at the poles is lesser than at the equator.
Thus, the weight of a person increases while moving from the equator to the poles.
• The Coriolis force increases with an increase in latitude. Thus, the value of Coriolis force is higher at
the poles than at the equator.
• Hence, both statements I and statement II are individually correct but statement II is not the
correct explanation for statement I. Hence, option (b) is the correct answer.

Q 95.D
• Saturated Fatty Acids:
o Saturated fatty acids are a type of fat molecule that consists of carbon atoms bonded to hydrogen
atoms with single bonds, resulting in a saturated structure.
o They are typically solid at room temperature and are commonly found in animal-based foods such
as meat, dairy products (butter, cheese), and some plant-based sources like coconut oil and palm
oil.
o Saturated fats have been traditionally associated with an increased risk of cardiovascular disease due
to their ability to raise LDL (low-density lipoprotein) cholesterol levels in the blood.
o Foods high in saturated fats are often linked to conditions like heart disease, obesity, and stroke.
• Unsaturated Fatty Acids:
o Unsaturated fatty acids are a type of fat molecule that contains one or more double bonds between
carbon atoms in the carbon chain, resulting in fewer hydrogen atoms and a bent or "kinked" structure.
o Unsaturated fats are found in both plant-based and animal-based foods. While some animal-
based foods like fatty fish contain omega-3 fatty acids, plant-based sources such as nuts, seeds,
avocados, and vegetable oils are rich sources of unsaturated fats as well. Hence statement 2 is not
correct.
o Omega-3 and omega-6 fatty acids are essential polyunsaturated fatty acids that play crucial
roles in various bodily functions, including brain function, inflammation regulation, and cell
membrane structure. Hence statement 1 is not correct.
• Trans fats:
o Trans fats are a type of unsaturated fatty acid with a trans configuration, and they are harmful to
health, particularly cardiovascular health. While they share some properties with saturated fats, trans
fats are distinct in their molecular structure and health effects, and they are widely recognized as one
of the most harmful dietary fats.
o Transfats actually increase the tendency of blood to clot. They can activate and aggregate
platelets, leading to an increased risk of thrombosis, which is the formation of blood clots. Hence
statement 3 is not correct.

41 www.visionias.in ©Vision IAS

Telegram: UPSC_SARTHI
Q 96.C
• The Ganga sub-basin extends over an area of 1086000 sq. km and lies in India, Tibet, Nepal, and
Bangladesh. The drainage area lying in India is 861404 sq. km which is nearly 26.2% of the total
geographical area of the country.
• The sub-basin is bounded on the north by the Himalayas, on the west by the Aravallis and the ridge
separating it from the Indus basin, on the south by the Vindhyas and Chhotanagpur plateaus, and on the
east by the Brahmaputra ridge.
• The Ganga basin has a large number of water resource assets and surface water bodies which is covered
by 11 states and most of its area is covered by Uttar Pradesh (28.02%) and Madhya Pradesh (21.02%) of
the total basin area.
• The other states in the embrace of the Ganga basin are Rajasthan, Bihar, Jharkhand, Chhattisgarh,
West Bengal, Uttarakhand, Haryana, Himachal Pradesh and Delhi. The State-wise distribution of the
drainage area is given in the below chart:

• Thus, option (c) is the correct answer.

Q 97.A
• The Strait of Dover, often referred to as the Dover Strait, is a narrow and historically significant
waterway that separates southern England (United Kingdom) from northern France.
o It serves as a natural channel connecting the English Channel to the North Sea.
o The strait is known for its strong tidal currents and is a vital maritime passage for ferry and shipping
routes between the UK and continental Europe.
• The Strait of Bosphorus, located in Istanbul, Turkey, connects the Black Sea to the Sea of
Marmara.
o This natural waterway is of great importance as it marks the entrance to the Mediterranean Sea from
the Black Sea.
o It plays a crucial role in maritime trade and is known for its stunning views, with Istanbul's historic
sites lining its shores.
• The Strait of Hormuz is a strategically significant waterway situated between the United Arab
Emirates and Oman on one side and Iran on the other.
o It connects the Persian Gulf to the Gulf of Oman and the Arabian Sea.
o The strait is renowned for its geopolitical importance, as it is a critical choke point for global oil
shipping, with a substantial portion of the world's oil supply passing through it.
• Cook Strait is located between New Zealand's North Island and South Island.
o It serves as a vital waterway within New Zealand, connecting the Tasman Sea to the south with the
South Pacific Ocean to the north.
o The strait is characterized by strong tidal flows and is important for maritime transportation and trade
within the country.
42 www.visionias.in ©Vision IAS

Telegram: UPSC_SARTHI
• Hence option (a) is the correct answer.

Q 98.B
• Recently, the Bihar government has decided to accord permission for exploration of gold reserves in
Jamui district.
• Overview of the Gold industry in India
o Total gold reserves in India are more than 500 million tonnes as per national mineral inventory data
o India is the world's second-biggest consumer of gold, after China. India accounts for 25% of global
gold demand. Hence statement 1 is not correct.
• India imports nearly 89% of its demand (According to a NITI Aayog report). Hence statement 2 is
correct.
• Major gold mines are East Singbhum (Jharkhand), Raichur and Kolar (Karnataka), and Chittoor ( Andra
Pradesh).
• Mines and Minerals (Development and Regulation) Act, 1957 is a regime for granting mineral
concessions through auction.
• National Minerals Exploration Policy (NMEP), 2016 aims to stimulate non-fuel and non-coal mining
exploration by allowing private companies to enter into a transparent bidding process, conducted via e-
auction.
• 100% FDI is permitted under the automatic route in this sector.
• Gold mining in India dates back to the first millennium BC and throughout the 20th century was
dominated by the Kolar Gold Field (KGF). KGF is one of the world's deepest gold mines at a depth of
3000 meters, the government has decided to revive the gold mining here.

Q 99.B
• The World’s first prototype of the BS 6 Stage II ‘Electrified Flex fuel vehicle’ was launched. Toyota
recently unveiled a prototype of the Innova Hycross with a flex-fuel hybrid powertrain, its first car in
India with this option, and one that the Japanese carmaker claims is the world’s first BS6 Stage II-
compliant flex-fuel vehicle. Toyota had displayed an imported Corolla flex-fuel hybrid sedan as a pilot
project. The pilot was initiated as part of a government-led push to commercially deploy this technology,
which is already in use in markets such as Brazil, Canada, and the United States.
• Flex Fuel Vehicles (FFVs) are designed with internal combustion engines capable of running on
different types of fuel or a mixture of fuels, such as petrol and ethanol. Hence statement 1 is correct.
• It has both a flex-fuel engine as well as an electric powertrain. Offers higher use of ethanol combined
with better fuel efficiencies. Hence statement 2 is correct.
• FFVs have a lower mileage compared to traditional gasoline vehicles. Hence statement 3 is not
correct.
• The advantages are lower harmful pollutant emissions like Carbon monoxide, sulphur, etc., and reduced
import dependence on crude oil.

43 www.visionias.in ©Vision IAS

Telegram: UPSC_SARTHI
• The transition to Bharat Stage VI (BS VI) emission standards is a crucial step in regulating air
pollutants from internal combustion and spark-ignition engines. India made a significant move, leaping
directly from BS-IV to BS-VI norms effective from April 01, 2020. This shift brought about a
remarkable improvement in fuel quality by reducing permissible sulfur content by 80%, from 50 ppm to a
maximum of 10 ppm.
• The upgraded fuel quality enabled the integration of advanced emission control technologies,
including Diesel Particulate Filters (DPF) for minimizing Particulate Matter (PM) emissions and
Selective Catalyst Reduction (SCR) systems to reduce Nitrogen Oxide (NOx) emissions. This strategic
move aligns with a commitment to environmental sustainability and the adoption of cleaner technologies
in the automotive sector.

Q 100.C
• India’s Moon mission Chandrayaan-3 scripted history by successfully landing on the lunar surface at 6:04
pm on August 23. With the Lander accomplishing a ‘soft landing’ on the Moon’s south pole, India
becomes the only country to have ever done so.
• According to ISRO, the mission’s three objectives were to demonstrate a safe and soft landing on the
lunar surface, to demonstrate a Rover roving on the Moon and to conduct in-situ scientific experiments.
Soft landing simply means landing at a gentle, controlled speed to not sustain damage to a spacecraft.
• The critical technical maneuver that the Chandrayaan-3 Lander had to perform on August 23, when it
entered the final 15 minutes of its attempt to make a soft landing on the Moon, was to transfer its high-
speed horizontal position to a vertical one — in order to facilitate a gentle descent on to the surface.
• After Chandrayaan-2 failed in its soft landing mission, K Sivan, then chairman of ISRO, described this
process as “15 minutes of terror” for them. It includes four phases:
• The Rough Braking phase includes reducing the lander’s horizontal velocity from a range of 1.68
km/sec (more than 6,000 km/h) at a height of 30 km from the lunar surface to almost zero for a soft
landing at the designated site. This has to be done with precision, within certain durations.
• At a height of 7.42 km from the surface, the lander is to go into an “attitude hold phase” lasting around 10
seconds, during which it should tilt from a horizontal to a vertical position while covering a distance of
3.48 km.
• The “fine braking phase” lasts around 175 seconds, during which the lander is to move fully into a
vertical position. It is to traverse the final 28.52 km to the landing site, the altitude will come down to
800-1,000 m, and it would reach a nominal speed of 0 m/sec. It was between the “attitude hold phase” and
the “fine braking phase” that Chandrayaan-2 lost control and crashed.
• “Terminal descent” is the final stage, when the spacecraft is supposed to descend totally vertically
onto the surface. Hence, option (c) is the correct answer.

Copyright © by Vision IAS


All rights are reserved. No part of this document may be reproduced, stored in a retrieval system or
transmitted in any form or by any means, electronic, mechanical, photocopying, recording or otherwise,
without prior permission of Vision IAS.

44 www.visionias.in ©Vision IAS

Telegram: UPSC_SARTHI

You might also like